79
FEBRUARY 2020 PENNSYLVANIA BAR EXAMINATION Essay Questions and Examiners’ Analyses and Performance Test Pennsylvania Board of Law Examiners 601 Commonwealth Avenue, Suite 3600 P.O. Box 62535 Harrisburg, PA 17106-2535 (717) 231-3350 www.pabarexam.org ©2020 Pennsylvania Board of Law Examiners

FEBRUARY 2020 PENNSYLVANIA BAR EXAMINATION · PENNSYLVANIA BAR EXAMINATION Essay Questions and Examiners’ Analyses and Performance Test Pennsylvania Board of Law Examiners 601 Commonwealth

  • Upload
    others

  • View
    6

  • Download
    3

Embed Size (px)

Citation preview

Page 1: FEBRUARY 2020 PENNSYLVANIA BAR EXAMINATION · PENNSYLVANIA BAR EXAMINATION Essay Questions and Examiners’ Analyses and Performance Test Pennsylvania Board of Law Examiners 601 Commonwealth

FEBRUARY 2020

PENNSYLVANIA BAR

EXAMINATION

Essay Questions and Examiners’ Analyses

and

Performance Test

Pennsylvania Board of Law Examiners 601 Commonwealth Avenue, Suite 3600

P.O. Box 62535 Harrisburg, PA 17106-2535

(717) 231-3350 www.pabarexam.org

©2020 Pennsylvania Board of Law Examiners

Page 2: FEBRUARY 2020 PENNSYLVANIA BAR EXAMINATION · PENNSYLVANIA BAR EXAMINATION Essay Questions and Examiners’ Analyses and Performance Test Pennsylvania Board of Law Examiners 601 Commonwealth

i

Table of Contents Index ..................................................................................................................................................ii Question No. 1: Facts and Interrogatories, Examiner's Analysis and Grading Guidelines .............. 1 Question No. 2: Facts and Interrogatories, Examiner's Analysis and Grading Guidelines .............. 9 Question No. 3: Facts and Interrogatories, Examiner's Analysis and Grading Guidelines ..............15 Question No. 4: Facts and Interrogatories, Examiner's Analysis and Grading Guidelines ..............23 Question No. 5: Facts and Interrogatories, Examiner's Analysis and Grading Guidelines ..............32 Question No. 6: Facts and Interrogatories, Examiner's Analysis and Grading Guidelines ..............41 Performance Test and Grading Guidelines ........................................................................................50

Page 3: FEBRUARY 2020 PENNSYLVANIA BAR EXAMINATION · PENNSYLVANIA BAR EXAMINATION Essay Questions and Examiners’ Analyses and Performance Test Pennsylvania Board of Law Examiners 601 Commonwealth

Index

Question No. 1 1. Decedent’s Estates – Testamentary capacity 2. Decedent’s Estates – In terrorem clause 3. Professional Responsibility – Candor toward the tribunal 4. Federal Income Tax – Home office deduction Question No. 2 1. Business Organizations – Dissenters rights 2. Business Organizations – Effect of merger on real property & debts 3. UCC Article II – Gap filler delivery 4. UCC Article II – Express warranty sample Question No. 3 1. Criminal Law – Receiving stolen property 2. Criminal Procedure – Search & seizure exceptions to warrant requirement 3. Evidence – Hearsay – former testimony exception 4. Family Law – Marital property Question No. 4 1. Constitutional Law – Procedural Due Process 2. Employment Law – Prima facie case ADA 3. Civil Procedure – Discovery work product doctrine

ii

Page 4: FEBRUARY 2020 PENNSYLVANIA BAR EXAMINATION · PENNSYLVANIA BAR EXAMINATION Essay Questions and Examiners’ Analyses and Performance Test Pennsylvania Board of Law Examiners 601 Commonwealth

Question No. 5

1. Property – Equitable conversion & risk of loss 2. Contracts – Consideration 3(a). Property – General warranty deed 3(b). Property – Special warranty deed 4(a). Contracts – Parol evidence rule 4(b). Contracts – Parol evidence rule mutual mistake exception Question No. 6 1. Torts – Design defect & failure to warn defect 2. Civil Procedure – Joinder additional defendant 3. Evidence – Hearsay – business records exception

iii

Page 5: FEBRUARY 2020 PENNSYLVANIA BAR EXAMINATION · PENNSYLVANIA BAR EXAMINATION Essay Questions and Examiners’ Analyses and Performance Test Pennsylvania Board of Law Examiners 601 Commonwealth

Question No. 1

Jack is a 92-year-old, single man who resides in B County, Pennsylvania. Jack has three

adult children, Amy, Ben, and Cindy. Amy and Ben live outside the state and do not visit often. His

daughter, Cindy, lives near Jack in B County and sees him frequently.

In December 2018, Jack completed an estate planning questionnaire on DraftYourWill.com,

a popular website that allows non-lawyers to draft their own legal documents for a modest fee. The

online program asked Jack to identify all of his children, whether or not he intended to name them as

beneficiaries. Jack identified Amy and Cindy but not Ben. Next, the program instructed Jack to

provide complete information for all assets, including real estate, bank accounts, and investments.

Jack listed only his checking account at Big Bank, which held $100,000 in cash. He forgot to report

his investment account with Friendly Financial, which held $5,000,000 of stocks and bonds. Both

accounts were owned by Jack in his individual name.

Jack stated on the DraftYourWill.com questionnaire that he wanted to leave “80% of his

estate to Cindy and 20% to Amy.” Jack named Big Bank as executor. He elected to include a “No

Contest Clause,” and adopted the sample language that stated, “If any beneficiary challenges the

validity of this will or any gift made hereunder, he or she shall forfeit his or her share.” The program

required Jack to verify that “all of the information provided is true and correct to the best of my

knowledge.” DraftYourWill.com then generated a will for disposition of Jack’s estate as set forth

above and containing the “No Contest Clause” language. Jack printed and properly signed the will

on December 15, 2018.

In January 2019, Cindy decided to convert the basement of her house into a home office for

her successful sales business. She outfitted the space with a desk and computer. A frequent

participant in online cycling classes, Cindy also placed her exercise bike in the basement. During

1

Page 6: FEBRUARY 2020 PENNSYLVANIA BAR EXAMINATION · PENNSYLVANIA BAR EXAMINATION Essay Questions and Examiners’ Analyses and Performance Test Pennsylvania Board of Law Examiners 601 Commonwealth

2019, Cindy did not use any office other than the one in her basement, and she worked an average of

30 hours a week and exercised for an average of 7 hours a week in the home office.

On January 5, 2020, Jack died. Jack’s December 15, 2018, will was admitted to probate by

the Register of Wills. Amy and Ben filed a timely and procedurally proper petition with the

Orphan’s Court seeking to have Jack’s 2018 will invalidated. The petition argued that Jack lacked

mental capacity to make a will and, since there was no other will, the estate should pass to Amy,

Ben, and Cindy in equal shares pursuant to intestacy law. Cindy retained her close friend Laura, a B

County attorney, to represent her in the case and to seek to uphold the will.

During discovery, medical records were produced showing that, in 2017, Jack had been

diagnosed with Alzheimer's disease, a progressive brain disorder characterized by dementia and

deterioration of memory and cognitive function. The records noted specific examples of cognitive

decline in 2018. Jack’s completed DraftYourWill.com questionnaire was also produced.

1. How is the court likely to rule on the petition to invalidate Jack’s 2018 will?

2. Assume for this question only that Cindy prevails in having the will upheld, and she properly files a petition with the court arguing that Amy should be divested of her share of the estate. Will the “No Contest Clause” cause Amy to forfeit her share under the Pennsylvania Probate, Estates and Fiduciaries Code?

3. Assume for this question only, that while preparing for a hearing in the will contest,

Cindy tells Laura, who plans to call her to testify, that she plans to testify that she was present when her father signed the will and that Jack understood exactly what he was doing. Laura believes that, on the date in question, she and Cindy were on vacation together in Jamaica and confirms this by checking her personal calendar and Cindy’s

social media account. Under the Pennsylvania Rules of Professional Conduct, how should Laura proceed with respect to Cindy’s planned testimony?

4. For federal income tax purposes, Cindy is a cash-basis, calendar-year taxpayer and

itemizes deductions. Cindy computed the portion of her home expenses allocable to her basement office, and claimed a deduction for those costs on her 2019 federal income tax return. Under the Internal Revenue Code relating to business expenses, was Cindy permitted to deduct these expenses?

2

Page 7: FEBRUARY 2020 PENNSYLVANIA BAR EXAMINATION · PENNSYLVANIA BAR EXAMINATION Essay Questions and Examiners’ Analyses and Performance Test Pennsylvania Board of Law Examiners 601 Commonwealth

Question No. 1: Examiner’s Analysis

1. The court is likely to rule that Jack lacked testamentary capacity.

The Pennsylvania Probate, Estates and Fiduciaries (PEF) Code provides that “[a]ny person

18 or more years of age who is of sound mind may make a will.” 20 Pa.C.S.A. § 2501. “Capacity relates to soundness of mind . . . .” In Re Estate of Olshefski, 11 A.2d 487, 488 (Pa. 1940) (internal quotations and citation omitted). When a will is properly executed, a presumption of testamentary capacity arises. Id. “[M]ental competency is presumed and . . . the burden of establishing incompetency rests upon those who challenge competency.” In Re Hunter Will, 205 A.2d 97, 102 (Pa. 1964).

Testamentary capacity should be determined as close as possible to the time the testator

executed the will. In Re Estate of Ziel, 359 A.2d 728, 732 (Pa. 1976). The test for testamentary capacity is as follows:

Testamentary capacity exists when the testator has intelligent knowledge of the natural objects of his or her bounty, the general composition of the estate, and what he or she wants done with it, even if memory is impaired by age or disease . . . .

In Re Estate of Reichel, 400 A.2d 1268, 1270 (Pa. 1979). “[T]he testator need not have the ability to conduct business affairs.” Id.

A finding that a testator suffered from Alzeheimer’s disease does not, by itself, establish that

a testator lacked testamentary capacity. In re Angle, 777 A.2d 114, 123 (Pa. Super 2001). Where the testator’s mind was clear at times but unclear at other times, the question is the testator’s state of

mind at the time the will was executed. Aggas v. Munnell, 152 A. 840, 843 (Pa. 1930).

Therefore, Jack’s diagnosis of Alzeheimer’s disease does not conclusively establish that he

lacked testamentary capacity. The essential question is whether Jack possessed capacity at the time that he executed the will. The will was properly executed and a presumption of capacity arises. However, it appears that Amy and Ben as the will’s contestants will be able to show that Jack lacked testamentary capacity at the time Jack executed the will under the above test.

To know the natural objects of the testator’s bounty means that the testator understands the identity of the relatives or persons who will share in the estate. In Re Estate of Ballas, 2010 Pa. D. & C. Dec. LEXIS 552 at *27 (Pa Com. Pl. 2010), aff’d, 31 A.3d 742 (Pa. Super. 2011); see also Natural objects of bounty, BALLENTINE'S LAW DICTIONARY (3rd ed. 2010) (defining “natural objects of bounty” as “[t]hose members of a testator's family entitled on the face of things to his bounty”). As one of Jack’s three children, Ben is one of the natural objects of Jack’s bounty. The program asked Jack to identify all of his children, whether or not he intended to include them as beneficiaries. Because he did not identify Ben, there is a question of whether Jack was capable of identifying the natural objects of his bounty, one of the essential elements of testamentary capacity.1

1 Note that under Pennsylvania law, a testator may disinherit a child and Jack was not required to include Ben as a beneficiary in his will. However, the online questionnaire specifically asked Jack to identify all children regardless of whether they were to be included as a beneficiary.

3

Page 8: FEBRUARY 2020 PENNSYLVANIA BAR EXAMINATION · PENNSYLVANIA BAR EXAMINATION Essay Questions and Examiners’ Analyses and Performance Test Pennsylvania Board of Law Examiners 601 Commonwealth

Additionally, while the testator need not know every asset he possesses, he must have general

knowledge of the property of which the estate consists. Estate of Ballas, 2010 Pa. D. & C. at *27. Jack was asked detailed questions about his finances, and he identified only his bank account. He failed to identify his significant investment account, which is much greater in value than his bank account and represented the majority of his estate. Jack did not seem to understand the composition of his estate, and Amy and Ben will likely be able to show that Jack lacked testamentary capacity.

Because it appears from Jack’s answers to the estate planning questionnaire that he did not

know the natural objects of his bounty or the contents of his estate, it is likely that the court will find that Jack lacked testamentary capacity. 2. The court will consider whether Amy had probable cause to challenge the will and will

likely not enforce the No Contest clause.

Jack’s will contained a “no contest” or “in terrorem” clause, which provided in relevant part

that if any beneficiary challenged the validity of the instrument, he or she would forfeit his or her share of the estate. Therefore, under the express terms of the will, because Amy brought a challenge to the will, her share of the estate may be subject to forfeiture.

“No contest” clauses are valid in Pennsylvania because they promote “the favorable public

policy of limiting costly, time-consuming litigation against the estate when such litigation is not founded upon probable cause but rather upon disappointment over [the] amount received.” In re

Estate of Simpson, 595 A.2d 94, 100 (Pa. Super. 1991). However, the PEF Code provides that a “no

contest” clause in a will shall not be enforced if there was probable cause for the will challenge: A provision in a will or trust purporting to penalize an interested person for contesting the will or trust or instituting other proceedings relating to the estate or trust is unenforceable if probable cause exists for instituting proceedings.

20 Pa.C.S.A. § 2521. The statute, enacted in 1994, codified existing Pennsylvania law. In Re Estate

of Powell, 209 A.3d 373, 377 (Pa. Super. 2019).

[T]he penalty of forfeiture of the gift or devise ought not to be imposed when it clearly appears that the contest to have the will set aside was justified under the circumstances, and was not the mere vexatious act of a disappointed child or next of kin.

In re Estate of Friend, 58 A. 853, 854 (Pa. 1904). In Friend, the testator’s son challenged his

mother’s will on the basis of lack of testamentary capacity. Id. at 856. The court held that the decedent was clearly possessed of the requisite capacity. Id. However, because the evidence showed that the testator had been in a “distressing nervous condition” and apparently had favored

one son above other children, including the challenger, the court held that the challenger had probable cause to institute his action. Id. at 856-57.

4

Page 9: FEBRUARY 2020 PENNSYLVANIA BAR EXAMINATION · PENNSYLVANIA BAR EXAMINATION Essay Questions and Examiners’ Analyses and Performance Test Pennsylvania Board of Law Examiners 601 Commonwealth

Here, it is likely that Amy had probable cause to challenge Jack’s testamentary capacity at the time he executed the will. Jack was 92-years-old and had been diagnosed with Alzheimer’s disease. At the time he executed the will, Jack failed to remember and name one of his children and evidently forgot about $5,000,000 in assets. It is highly likely that the will contest was supported by probable cause, and, consequently, Amy will not forfeit her share of her father’s estate. Therefore, even if the petition to invalidate the will was denied, the court will likely find that there was probable cause for the will challenge, and will not enforce the “no contest” clause. 3. To comply with the RPC’s requirement of candor towards a tribunal, Laura must not

offer evidence she knows to be false.

The Rules of Professional Conduct (“RPC”) require attorneys to act with candor towards the

tribunal. Pa. R.P.C. 3.3. Specifically, the RPC provide that a lawyer shall not knowingly:

offer evidence that the lawyer knows to be false. If a lawyer, the lawyer’s client, or a

witness called by the lawyer, has offered material evidence before a tribunal or in an ancillary proceeding conducted pursuant to a tribunal’s adjudicative authority, such as a deposition, and the lawyer comes to know of its falsity, the lawyer shall take reasonable remedial measures, including, if necessary, disclosure to the tribunal. A lawyer may refuse to offer evidence, other than the testimony of a defendant in a criminal matter,that the lawyer reasonably believes is false.

Pa. R.P.C. 3.3(a)(3). The Rule states further that “a lawyer . . . who knows that a person intends to engage . . . in criminal or fraudulent conduct related to the proceeding shall take reasonable remedial measures, including, if necessary, disclosure to the tribunal.” Pa. R.P.C. 3.3(b).

“‘Tribunal’ denotes a court, an arbitrator in a binding arbitration proceeding or a legislative body, administrative agency or other body acting in an adjudicative capacity.” Pa. R.P.C. 1.0(m). Here, the will contest is proceeding before the Orphans’ Court, which clearly meets the definition of a tribunal. Cindy is Laura’s client as well as a witness she plans to call at the hearing.

Cindy’s testimony is material because, if true, it would offer evidence that Jack had testamentary capacity on the day he signed his will, and that the will was valid and should be upheld. Therefore, if Laura knows that Cindy’s testimony is false, she must take reasonable remedial measures.

According to the RPC, “knows” means “actual knowledge of the fact in question.” Pa. R.P.C.

1.0(f).

The prohibition against offering false evidence only applies if the lawyer knows that the evidence is false. A lawyer’s reasonable belief that evidence is false does not preclude its presentation to the trier of fact. A lawyer’s knowledge that evidence is

false, however, can be inferred from the circumstances. See Rule 1.0(f). Thus, although a lawyer should resolve doubts about the veracity of testimony or other evidence in favor of the client, the lawyer cannot ignore an obvious falsehood.

5

Page 10: FEBRUARY 2020 PENNSYLVANIA BAR EXAMINATION · PENNSYLVANIA BAR EXAMINATION Essay Questions and Examiners’ Analyses and Performance Test Pennsylvania Board of Law Examiners 601 Commonwealth

Pa. R.P.C. 3.3 cmt. 8. Laura was with Cindy outside the United States on the date in question. Laura confirmed her memory of this by consulting her own personal records and Cindy’s social

media account. Since Laura has confirmed these facts to be true, the testimony Cindy plans to give is false. If Laura was unsure whether the testimony was true, she could decide whether to offer it as evidence. However, because Laura has actual knowledge that the evidence is false, she is prohibited from introducing it before the tribunal.

If a lawyer knows that the client intends to testify falsely or wants the lawyer to introduce false evidence, the lawyer should seek to persuade the client that the evidence should not be offered. If the persuasion is ineffective and the lawyer continues to represent the client, the lawyer must refuse to offer the false evidence. If only a portion of a witness's testimony will be false, the lawyer may call the witness to testify but may not elicit or otherwise permit the witness to present the testimony that the lawyer knows is false.

Pa. R.P.C. 3.3 cmt. 6. Laura should therefore speak with Cindy and attempt to persuade her not to offer the false testimony. If she is unsuccessful in convincing Cindy, at the hearing, she must refuse to elicit Cindy’s testimony about the date the will was signed. If she is aware that Cindy intends to present fraudulent testimony, she will be required to take remedial measures, possibly including disclosure to the tribunal under Rule 3.3(b). Alternatively, Laura may need to withdraw from representation:

Normally, a lawyer’s compliance with the duty of candor imposed by this Rule does

not require that the lawyer withdraw from the representation of a client whose interests will be or have been adversely affected by the lawyer’s disclosure. The

lawyer may, however, be required by Rule 1.16 to seek permission of the tribunal to withdraw if the lawyer’s compliance with this Rule’s duty of candor results in such an

extreme deterioration of the client-lawyer relationship that the lawyer can no longer competently represent the client. Also see Rule 1.16(b) for the circumstances in which a lawyer will be permitted to seek a tribunal’s permission to withdraw. In

connection with a request for permission to withdraw that is premised on a client’s

misconduct, a lawyer may reveal information relating to the representation only to the extent reasonably necessary to comply with this Rule or as otherwise permitted by Rule 1.6.

Pa. R.P.C. 3.3 cmt. 15. Rule 1.16 provides that a lawyer must withdraw from representation of a client if the representation will result in violation of the RPC. Pa.R.P.C. 1.16(a)(1); see also

Pa.R.P.C. 1.2(d).

Therefore, if Laura is unable to convince Cindy not to testify falsely, and she does not believe she can proceed with the case without violating Rule 3.3, she should consider whether she must disclose the false testimony to the court, or withdraw as counsel. 4. Cindy was not permitted by the Internal Revenue Code to take a deduction for the

expenses related to her home office.

6

Page 11: FEBRUARY 2020 PENNSYLVANIA BAR EXAMINATION · PENNSYLVANIA BAR EXAMINATION Essay Questions and Examiners’ Analyses and Performance Test Pennsylvania Board of Law Examiners 601 Commonwealth

“[O]rdinary and necessary expenses paid or incurred in carrying on [a] trade or business” are generally deductible. 26 U.S.C.S. § 162. However, except in certain limited circumstances, a taxpayer cannot deduct expenses for use of a dwelling unit. 26 U.S.C.S. § 280A(a).

Section 280A also provides exceptions permitting deductions for business expenses related to

a dwelling unit. That section provides in relevant part the following: [T]o the extent such item is allocable to a portion of the dwelling unit which is exclusively used on a regular basis— (A) as the principal place of business for any trade or business of the taxpayer . . . For purposes of subparagraph (A), the term “principal place of business” includes a

place of business which is used by the taxpayer for the administrative or management activities of any trade or business of the taxpayer if there is no other fixed location of such trade or business where the taxpayer conducts substantial administrative or management activities of such trade or business.

26 U.S.C.S. § 280A(c)(1).

“Exclusive use of a portion of a taxpayer's dwelling unit means that the taxpayer must use a specific part of a dwelling unit solely for the purpose of carrying on his trade or business.” Hefti v.

Commissioner, T.C. Memo 1993-128, *7 (discussing the legislative history of I.R.C. § 280A and proposed Treasury Regulations). In addition, the use of the home office must be regular, it cannot be occasional or incidental. Id. at *8

In Hefti, the Tax Court affirmed the Internal Revenue Service’s denial of a claimed home

office deduction where the evidence showed that the alleged home office was not used exclusively for business purposes. Id. at *8-10.

Here, Cindy fails to meet the requirements necessary to claim a home office deduction. Her home office likely qualifies as a principal place of business because she does not maintain a separate office. However, the facts state that Cindy spends seven hours per week exercising in the home office. While she appears to use the home office regularly for business, because Cindy spends a portion of her time exercising there, it is clear she does not use it exclusively for business purposes. As a result, Cindy should not have claimed expenses related to her home office as a deduction on her federal income tax return.

7

Page 12: FEBRUARY 2020 PENNSYLVANIA BAR EXAMINATION · PENNSYLVANIA BAR EXAMINATION Essay Questions and Examiners’ Analyses and Performance Test Pennsylvania Board of Law Examiners 601 Commonwealth

Question No. 1: Grading Guidelines

1. Testamentary capacity.

Comment: Applicants should discuss whether the testator satisfied the elements of testamentary capacity.

6 points.

2. In terrorem clause.

Comment: Applicants should discuss Pennsylvania law limiting enforcement of an in terrorem clause where there is probable cause to challenge a will.

4 points.

3. Duty of candor towards tribunal.

Comment: Applicants should discuss the rule requiring candor towards a tribunal and the prohibition on offering evidence a lawyer knows to be false, as well as discuss possible remedial measures such as withdrawal and disclosure to the tribunal.

5 points.

4. Home office deduction.

Comment: Applicants should identify and apply the rules for deducting home office expenses and come to a well-reasoned conclusion.

5 points.

8

Page 13: FEBRUARY 2020 PENNSYLVANIA BAR EXAMINATION · PENNSYLVANIA BAR EXAMINATION Essay Questions and Examiners’ Analyses and Performance Test Pennsylvania Board of Law Examiners 601 Commonwealth

Question No. 2

Smith Hardware, Inc. (Smith), a Pennsylvania non-publicly traded corporation, operates a

general hardware store at a single location in Pennsylvania. Smith was founded in 1940 by John

Smith who was Smith’s sole shareholder until his death in 2000. Upon his death his certificated

shares in Smith passed equally to his three children, Art, Bill, and Carl, who each received

certificates for their shares. Art, Bill, and Carl have served as Smith’s board of directors since

their father’s death and Art and Bill have served as Smith’s president and secretary/treasurer,

respectively, since 2000. Carl moved to Colorado in 2001. Carl’s involvement with Smith since

2001 has been limited to telephone participation at monthly board meetings and the annual

shareholders’ meeting.

Smith has had difficulty competing with regional and national hardware chains.

Northeast Hardware, Inc. (NE), a regional hardware chain with stores in Pennsylvania, is Smith’s

most aggressive competitor. Late last year, NE’s CEO contacted Art and Bill and proposed that

NE acquire Smith via a merger. The proposed plan of merger would provide that Art, Bill, and

Carl would each receive one share of NE stock for each share of Smith stock that they own (a

one-for-one exchange of shares) and that Art and Bill would receive employment contracts with

NE for at least the next five years.

Art and Bill presented the NE merger plan at last month’s board meeting. Carl objected

to the proposed plan. Carl rightfully believes the plan and proposed share exchange would dilute

the value of his equity holdings since an NE share is worth less than a Smith share. Art and Bill

do not contest Carl’s valuation analysis but, nonetheless, believe, in their sound business

judgment, the merger is not fraudulent or fundamentally unfair and is the best course to follow

given the competitive pressures on Smith. Despite Carl’s strong objections, Art and Bill voted to

9

Page 14: FEBRUARY 2020 PENNSYLVANIA BAR EXAMINATION · PENNSYLVANIA BAR EXAMINATION Essay Questions and Examiners’ Analyses and Performance Test Pennsylvania Board of Law Examiners 601 Commonwealth

approve the merger plan at the directors meeting and have called for the scheduling of a

shareholders’ meeting to approve the merger (Smith’s bylaws require that a merger be approved

by a majority of its shareholders).

Art and Bill have raised questions regarding the effect of the merger upon the real estate

owned by Smith, the outstanding mortgage on the real estate, and the status of its other debts.

Last month, Art visited Garments, a manufacturer of various work clothes, to purchase

one-piece painter’s coveralls that Smith’s would then screen print or embroider with a painting

logo for its painting contractor customers. Art indicated that he needed bright white coveralls.

Garments’ salesperson indicated that bright white was not a problem and pulled a sample of

bright white coveralls from a shelf and showed it to Art indicating that Garments could provide

similar coveralls to Smith. Art approved the sample coveralls and signed a valid and enforceable

purchase agreement with Garments on behalf of Smith to purchase 500 bright white coveralls at

$10 per piece. The purchase agreement properly disclaimed all implied warranties arising under

the Pennsylvania Uniform Commercial Code (Code) but failed to address time of delivery.

1. Upon receipt of notice of the shareholders meeting scheduled to approve the merger, what rights does Carl have regarding the expected diminished value of his shares and how should he proceed prior to and at the meeting to protect his rights and with what likely result?

2. Without considering any contractual provisions in loan documents that might be implicated, under Pennsylvania law governing business organizations, if the merger is approved and consummated, what effect, if any, would it have upon ownership of the Smith real estate, the mortgage on that property, and the status of Smith’s other debts to its general creditors?

3. Under the Code, when must Garments deliver the coveralls? 4. If, upon delivery, the coveralls are gray in color rather than bright white, under

the Code could Smith assert a breach of warranty claim against Garments based on the coverall that Garment’s salesperson showed Art notwithstanding

Garments’ proper disclaimer of implied warranties?

10

Page 15: FEBRUARY 2020 PENNSYLVANIA BAR EXAMINATION · PENNSYLVANIA BAR EXAMINATION Essay Questions and Examiners’ Analyses and Performance Test Pennsylvania Board of Law Examiners 601 Commonwealth

Question No. 2: Examiner’s Analysis

1. Carl has dissenters rights under the Pennsylvania Business Corporation Law of

1988 (BCL). Carl must follow the steps required to assert those rights, and, if

properly exercised, should receive fair value for his shares from Smith.

Carl can exercise dissenters rights under the Pennsylvania Associations Code (Associations Code) and the BCL. Section 333(d)(1) of the Associations Code provides, “[e]xcept as provided in paragraph (2) if a shareholder of a domestic business corporation that is to be a merging association objects to the plan of merger and complies with Subchapter D of Chapter 15 (relating to dissenters rights), the shareholder shall be entitled to dissenters rights to the extent provided in that subchapter.” 15 Pa.C.S.A. § 333(d)(1); see also, 15 Pa.C.S.A. § 1571(a). The BCL provides certain exceptions to dissenter’s rights (e.g., national securities

exchange companies or companies with more than 2,000 shareholders), which are not applicable to the instant matter. 15 Pa.C.S.A. §1571(b)(1). Additionally, the bylaws of Smith require approval of a proposed merger by a majority of the shareholders of Smith, thereby eliminating possible exceptions that allow for mergers adopted by the directors without shareholder approval. See, e.g., 15 Pa.C.S.A. § 321(d).

If Carl follows the BCL procedural rules to exercise his dissenters rights he should be

entitled to a valuation of his shares and a payment from Smith. 15 Pa.C.S.A. § 333(d)(1); 15 Pa.C.S.A. § 1571. It should be noted that absent a showing of fraud or fundamental unfairness dissenters rights will be Carl’s exclusive remedy. 15 Pa.C.S.A. § 1105.

Carl must take the following actions to exercise dissenters rights: (1) “file with the

corporation, prior to the [shareholder vote on the proposed merger], a written notice of intention to demand that he be paid . . . fair value for his shares if the proposed action is effectuated[; (2)] effect no change in the beneficial ownership of his shares from the date of his [giving of notice] . . . continuously through the effective date of the proposed action[;] and [(3)] . . . refrain from voting his shares in approval of the proposed merger.” 15 Pa.C.S.A. § 1574. Section 1575 of the BCL further provides, “If the proposed corporate action is approved by the required vote at a meeting of shareholders of a business corporation, the corporation shall deliver a further notice to all dissenters who gave due notice of intention to demand payment of the fair value of their shares and who refrained from voting in favor of the proposed action.” 15 Pa.C.S.A. § 1575(a). In the case of a dissenter holding certificated shares (like Carl), the notice is to indicate where and when a demand for payment must be sent and where the share certificates held by the dissenter should be deposited in order to get payment. The time set for receipt of the demand and deposit of the certificated shares cannot be less than 30 days from the mailing of the notice. 15 Pa.C.S.A. § 1575. Thus, upon receipt of the notice from the corporation Carl should make the required demand, deliver his shares, and await the payment of fair value of his shares by Smith. It should be noted that the BCL provides mechanisms for the dissenter to challenge the value assigned to the shares and a dispute resolution process to be followed to resolve contested fair value and payment issues. See, 15 Pa.C.S.A. §§ 1577, 1578, 1579, and 1580. If Carl takes the preceding steps, he should receive fair value for his shares.

11

Page 16: FEBRUARY 2020 PENNSYLVANIA BAR EXAMINATION · PENNSYLVANIA BAR EXAMINATION Essay Questions and Examiners’ Analyses and Performance Test Pennsylvania Board of Law Examiners 601 Commonwealth

2. The merger will vest title to the real property in the surviving corporation; with NE,

the surviving corporation, taking the property subject to the mortgage, and Smith’s

debts shall also pass to NE.

Section 336 of the Associations Code addresses the effect of merger. See 15 Pa.C.S.A. § 336. Three questions are asked regarding this proposed merger. First, what effect will the merger have upon the real estate owned by Smith? Section 336(a)(3) provides, “[a]ll property of

each merging association vests in the surviving association without reversion or impairment, and the merger shall not constitute a transfer of any of that property.” In other words, the surviving corporation becomes the owner of all real property of the merged associations. Thus, the real estate will become property of NE, the surviving corporation.

Second, what effect will the merger have upon the mortgage on Smith’s real property? Section 336(a)(7) provides, “[l]iens on the property of the merging association shall not be impaired by the merger.” Thus, the real estate held by NE shall be held subject to the mortgage.

Finally, what effect will the merger have upon Smith’s debt owed to general creditors? Section 336(a)(4) provides, “[a]ll debts, obligations and other liabilities of each merging association are debts, obligations and other liabilities of the surviving association.” Thus,

Smith’s debts shall be the debts and obligations of NE. 3. Under the Pennsylvania Uniform Commercial Code, Garments will be required to

deliver the coveralls within a reasonable time taking into account the circumstances

of the parties.

Article II of the Pennsylvania Uniform Commercial Code (Code) on Sales is generally applicable to transactions in goods. 13 Pa.C.S.A. § 2102. “Goods” are “all things . . . which are

movable at the time of identification to the contract for sale . . . .” 13 Pa.C.S.A. § 2105(a). The coveralls forming the subject matter of the valid contract between Smith and Garments are goods as they are movable objects, therefore, the Code would be applicable to the contract.

The Code contains sections that assist parties to a contract who have failed to address certain terms in the body of their agreement. These “gap filler” provisions “come in to play when the agreement of the parties . . . is silent” on the terms in question. See, James J. White

and Robert S. Summers, Uniform Commercial Code, § 3-4 (4th Ed. 1995). Garments and Smith have executed an agreement that is silent on when delivery of the

coveralls should occur. Section 2309 of the Code provides, “[t]he time for shipment or delivery

or any other action under a contract if not provided in this division or agreed upon shall be a reasonable time.” 13 Pa.C.S.A. § 2309(a). Accordingly, delivery must be made by Garments within a “reasonable time.” A “reasonable time” will be derived by examining the duties of

good faith and provisions relating to commercial reasonableness under the Code. A “reasonable

time” depends upon “what constitutes acceptable commercial conduct in view of the nature, purpose and circumstances of the action to be taken.” 13 Pa.C.S.A. § 2309 cmt. 1, see also 13 Pa.C.S.A. § 1205. Thus, Garments will be required to deliver the coveralls within a reasonable time given the circumstances of the parties. Industry standards and common delivery times will

12

Page 17: FEBRUARY 2020 PENNSYLVANIA BAR EXAMINATION · PENNSYLVANIA BAR EXAMINATION Essay Questions and Examiners’ Analyses and Performance Test Pennsylvania Board of Law Examiners 601 Commonwealth

be considered and will provide guidance in establishing what will be a reasonable time. See

James J. White and Robert S. Summers, Uniform Commercial Code, §3-5 (4th Ed. 1995).

4. Smith can assert a claim of breach of express warranty based upon the bright white

sample of the coveralls provided by Garments prior to Smith placing the order for

the suits and the claim would not be barred by the disclaimer of implied warranties.

The facts indicate that Garments properly disclaimed all implied warranties arising under the Code. Under Section 2316 of the Code, however, it is possible to disclaim implied warranties and still be found to have made an express warranty regarding the goods sold. Thomas M. Quinn, Quinn’s Uniform Commercial Code Commentary and Law Digest, Vol. 1 § 2-316[A] (2nd Ed. 1991).

Express warranties are addressed in Section 2313 of the Code. Section 2313(a)(3) provides, “[e]xpress warranties by the seller are created as follows: . . . (3) Any sample or model which is made a part of the basis of the bargain creates an express warranty that the whole of the goods shall conform to the sample or model.” 13 Pa.C.S.A. § 2313(a)(3). Subpart (b) of section 2313 further provides, “[i]t is not necessary to the creation of an express warranty that the seller use formal words such as ‘warrant’ or ‘guarantee’ or that he have a specific intention to make a

warranty . . . .” 13 Pa.C.S.A. § 2313(b).

Garment’s salesperson pulled coveralls off of a shelf and presented it to Art as a sample of the bright white coveralls that would be provided to Smith. After examining the sample Art, believing that Smith would be receiving similar bright white coveralls, signed a purchase order for the suits. Comment 6 to Section 2313 states, “[i]n general, the presumption is that any sample or model just as any affirmation of fact is intended to become a basis of the bargain. . . . If the sample has been drawn from an existing bulk, it must be regarded as describing values of the goods contracted for unless it is accompanied by an unmistakable denial of such responsibility.” 13 Pa.C.S.A. § 2313 cmt. 6. When a buyer was shown a sample of white newsprint to be used in books which he ordered and the completed books had gray pages, the goods did not conform to the sample and accordingly there was a breach of express warranty. See James J. White and Robert S. Summers, Uniform Commercial Code, §9-6 (4th Ed. 1995). Accordingly, Smith should have a valid claim for breach of express warranty under section 2313 of the Code.

13

Page 18: FEBRUARY 2020 PENNSYLVANIA BAR EXAMINATION · PENNSYLVANIA BAR EXAMINATION Essay Questions and Examiners’ Analyses and Performance Test Pennsylvania Board of Law Examiners 601 Commonwealth

Question No. 2: Grading Guidelines

1. Business organizations – dissenters rights arising from proposed merger

Comments: The candidates should discuss whether or not dissenters rights arise under the facts, the steps that must be taken to preserve and avail oneself of those rights, and with what likely result. 8 points 2. Business organizations – effect of merger on real estate, mortgage, and debts

Comments: The candidates should discuss the effect that the merger has upon real estate, a mortgage, and debts of the target. 3 points 3. Sales – gap filler addressing missing delivery term

Comments: The candidates should discuss the use of Code gap filler provisions to address a missing delivery term. 4 points 4. Sales – express warranty arising from sample

Comments: The candidates should discuss the presence of an express warranty arising from a sample provided to the buyer. 5 points

14

Page 19: FEBRUARY 2020 PENNSYLVANIA BAR EXAMINATION · PENNSYLVANIA BAR EXAMINATION Essay Questions and Examiners’ Analyses and Performance Test Pennsylvania Board of Law Examiners 601 Commonwealth

Question No. 3

In April of 2017 Mike purchased a Rocket 6, a 20-speed-racing bicycle, for $8,000. A

couple of months later, Mike met Sally and they married in July of 2019 in C County,

Pennsylvania, where the couple has continuously resided. A week after the marriage, Mike sold

his Rocket 6 for $5,000 and purchased a new Speedster X 20-speed-racing bicycle for $5,000.

Two weeks after he purchased the Speedster X, Mike’s mother gave him a new custom-

manufactured Dome bike helmet valued at $950 for his birthday.

In September of 2019, Mike decided he wanted to enter a 160-mile bike ride that was

being held in New Jersey. Mike asked his neighbor, Scott, if he had a bike rack he could borrow

to transport his bike to the New Jersey event. Scott responded: “This is your lucky day. I just

stole a bike rack off a car three weeks ago, and I decided I don’t want it. You can have it.” Mike

thanked Scott, took the rack, and used it to transport his bike to the New Jersey event.

A couple of days after returning from the New Jersey bike ride, Mike was feeling guilty

about keeping the bike rack. He decided to get rid of the bike rack by placing it at the curb with

his weekly garbage the night before the scheduled municipal pickup. The bike rack and garbage

were placed in a public area owned by the municipality. The next morning, before the garbage

truck arrived, Officer Lee saw the bike rack in front of Mike’s home beside Mike’s garbage cans.

The bike rack fit the description of one that had been reported stolen several weeks earlier.

Officer Lee immediately took the bike rack to the police station.

Based on further investigation, Officer Lee determined that the bike rack in front of

Mike’s house was stolen by Scott. Officer Lee subsequently conducted a proper interview of

Scott wherein Scott admitted he took the bike rack. Scott further informed Officer Lee that he

15

Page 20: FEBRUARY 2020 PENNSYLVANIA BAR EXAMINATION · PENNSYLVANIA BAR EXAMINATION Essay Questions and Examiners’ Analyses and Performance Test Pennsylvania Board of Law Examiners 601 Commonwealth

gave the bike rack to Mike and told Mike that he had stolen it. Officer Lee also lawfully spoke

with Mike who stated he placed the bike rack in front of his home to get rid of it.

Based upon Officer Lee’s investigation, Mike was charged with receiving stolen

property. At Mike’s preliminary hearing, where Mike was present and represented by counsel,

Scott testified that he told Mike that the rack was stolen when he gave it to him. Mike’s attorney

opted not to cross-examine Scott, although he had the opportunity to do so. At the conclusion of

the preliminary hearing, the court found that the Commonwealth had submitted sufficient

evidence to forward the receiving stolen property charge to trial. A stenographer was present at

the hearing and later produced a verbatim transcript of the hearing that she provided to the

district attorney (DA) and Mike’s attorney.

After the preliminary hearing, Scott was tragically killed in a boating accident. A week

later, Sally filed for divorce from Mike in the C County Court of Common Pleas.

1. For this question only assume the above facts are all admissible. Do the facts support a charge of receiving stolen property against Mike?

2. Mike’s attorney files a proper pre-trial motion requesting the suppression of the bike rack as evidence based on Mike’s right to be free from unreasonable search

and seizure under the United States and Pennsylvania Constitutions. Excluding the plain view doctrine, how would the court likely rule?

3. At Mike’s trial on the charge of receiving stolen property, the DA attempts to

have an officer read Scott’s preliminary hearing testimony to demonstrate that Scott had informed Mike that the bike rack was stolen and the defense objects to the introduction of this evidence by asserting that it is hearsay. How should the DA respond to the hearsay objection and how will the court likely rule?

4. Neither the Speedster X 20-speed-racing bicycle nor the helmet changed in value

from the date of the marriage through the divorce proceedings between Sally and Mike. What portion, if any, of the value of the Speedster X 20-speed-racing bicycle and helmet would be considered marital property for purposes of equitable distribution?

16

Page 21: FEBRUARY 2020 PENNSYLVANIA BAR EXAMINATION · PENNSYLVANIA BAR EXAMINATION Essay Questions and Examiners’ Analyses and Performance Test Pennsylvania Board of Law Examiners 601 Commonwealth

Question No. 3: Examiner’s Analysis

1. Assuming the facts are admissible they do support a charge of receiving stolen

property against Mike because he knew the property was stolen when he took

possession of it. “A person is guilty of theft if he intentionally receives, retains, or disposes of moveable

property of another knowing it has been stolen, or believing that it has probably been stolen, unless the property is received, retained, or disposed with intent to restore it to the owner.” 18 Pa. C.S.A. § 3925(a). The word “receiving” under Section 3925(a) “means acquiring possession, control or title, or lending on the security of the property.” 18 Pa. C.S.A. § 3925(b). “A person

‘knows’ that goods are stolen if he is ‘aware’ of that fact.” Commonwealth v. Newton, 994 A.2d 1127, 1132 (Pa. Super 2010) (citing 18 Pa. C. S. A. Section 302 (b)(2)(i)).

As applied here, Mike received the bike rack from Scott. A bike rack would be deemed

to be moveable property. See 18 Pa.C.S.A. § 3901. In his interview with Officer Lee, Scott stated that he told Mike the bike rack was stolen when he gave it to him. Scott also testified at the preliminary hearing that he told Mike the bike rack was stolen when he gave it to him. Moreover, Mike admitted to Officer Lee that he placed the bike rack in front of his house to get rid it. Thus, there are strong arguments that Mike knew that the bike rack was stolen at the time he received it as that fact was specifically communicated to him. There is no indication in the facts that Mike intended to restore the bike rack to its rightful owner or that he made any attempt to even determine the identity of the rightful owner. In fact, Mike took steps to discard the bike rack.

Based upon the facts presented, the charge of receiving stolen property is likely supported

against Mike.

2. The court would likely rule that Officer Lee’s seizure of the bike rack was

constitutional on the theory that the bike rack was abandoned by Mike and he no

longer had a reasonable expectancy of privacy in the bike rack. Thus, the bike rack

will be able to be used as evidence against Mike and it will be admissible at trial on

the charge of receiving stolen property. In Commonwealth vs. Vecchione, 476 A.2d 403, 408 (Pa. Super. 1984) the Superior

Court outlined the applicable law with respect to the issue of abandonment as follows:

It is hornbook law that abandoned property may be obtained and used for evidentiary purposes without regard to the existence of probable cause or a search warrant. Commonwealth v. Williams, 269 Pa. Super. 544, 410 A.2d 835 (1979). The theory of abandonment is predicated upon the clear intent of an individual to relinquish control of the property he possessed. Commonwealth v. Shoatz, 469 Pa. 545, 366 A.2d 1216 (1976). Personal belongings, such as Appellant’s

suitcase, “retain their constitutional protection until their owner meaningfully

abdicates control or responsibility.” Commonwealth v. Platou, 455 Pa. 258, 312 A.2d 29 (1973), cert. denied, 417 U.S. 976 (1974). The issue is not abandonment in the strict property right sense, but whether the person prejudiced by the search

17

Page 22: FEBRUARY 2020 PENNSYLVANIA BAR EXAMINATION · PENNSYLVANIA BAR EXAMINATION Essay Questions and Examiners’ Analyses and Performance Test Pennsylvania Board of Law Examiners 601 Commonwealth

had voluntarily discarded, left behind, or otherwise relinquished his interest in the property in question so that he could no longer retain a reasonable expectation of privacy with regard to it at the time of the search. Commonwealth v. Shoatz, supra 469 Pa. at 533, 366 A.2d at 1220 citing U.S. v. Colbert, 474 F.2d 174, 176 (5th Cir. 1973). Although abandoned property is admissible evidence, such property may not be utilized where the abandonment is coerced by unlawful police action. Commonwealth v. Harris, 491 Pa. 402, 421 A.2d 199 (1980); Commonwealth v. Jeffries, 454 Pa. 320, 311 A.2d 914 (1973); Commonwealth v.

Pollard, 450 Pa. 138, 299 A.2d 233 (1973).

In Commonwealth v. Shoatz, 366 A.2d 1216, 1220 (Pa. 1976), the Pennsylvania Supreme Court also observed as follows:

Abandonment is primarily a question of intent, and intent may be inferred from words spoken, acts done, and other objective facts. United States v. Cowan, 2d Cir. 1968, 396 F.2d 83, 87. All relevant circumstances existing at the time of the alleged abandonment should be considered. United States v. Manning, 5th Cir. 1971, 440 F.2d 1105, 1111.

In California v. Greenwood, 486 U.S. 35 (1988), the United States Supreme Court

addressed the warrantless search of trash which had been placed at curbside. The Court stated “the warrantless search and seizure of the garbage bags left at the curb outside the Greenwood

house would violate the Fourth Amendment only if respondents manifested a subjective expectation of privacy in their garbage that society accepts as objectively reasonably.” Id. at 39 (further citation omitted). The Court concluded as follows:

Here, we conclude that respondents exposed their garbage to the public sufficiently to defeat their claim to Fourth Amendment protection. It is common knowledge that plastic garbage bags left on or at the side of a public street are readily accessible to animals, children, scavengers, snoops and other members of the public. Moreover, respondents placed their refuse at the curb for the express purpose of conveying it to a third party, the trash collector, who might himself have sorted through respondents’ trash or permitted others, such as the police, to do so. Accordingly, having deposited their garbage in an area particularly suited for public inspection and, in a manner of speaking, public consumption, for the express purpose of having strangers take it, respondents could have had no reasonable expectation of privacy in the inculpatory items that they discarded. . . . Hence, what a person knowingly exposes to the public, even in his own home or office, is not a subject of Fourth Amendment protection.

Id. at 40 (internal citations, quotations, and footnotes omitted); accord Commonwealth v. Perdue, 564 A.2d 489, 492-93 (Pa. Super. 1989).

As applied here, the facts indicate that Mike placed the bike rack in front of his home, along with his other garbage, for municipal refuse pickup. He placed the bike rack in a public area, i.e. at the curb on municipal property. By placing the bike rack for garbage pickup in a public area, it can be inferred that Mike intended to relinquish control of it. This is further

18

Page 23: FEBRUARY 2020 PENNSYLVANIA BAR EXAMINATION · PENNSYLVANIA BAR EXAMINATION Essay Questions and Examiners’ Analyses and Performance Test Pennsylvania Board of Law Examiners 601 Commonwealth

supported by Mike’s statement to Officer Lee that he placed the bike rack in front of his house to get rid of it. When Mike placed the rack for municipal garbage pickup, it can be argued that he no longer retained a reasonable expectation of privacy with regard to that property. Further, there is no evidence whatsoever that the abandonment of the bike rack by Mike was in any way coerced by unlawful police action.

Because the bike rack will likely be deemed to have been abandoned by Mike and he no longer had a reasonable expectation of privacy in the bike rack, the court will probably rule its seizure was constitutional and permit its admission as evidence at Mike’s trial on the charge of receiving stolen property.

3. The DA should respond that, although Scott’s preliminary hearing testimony would

be deemed to be hearsay it should be admitted under the former testimony

exception to the hearsay rule. The court would likely agree and permit the

introduction of Scott’s preliminary hearing testimony.

Under Pa.R.E. 801(c) hearsay is defined as “a statement that (1) the declarant does not make while testifying at the current trial or hearing; and (2) a party offers in evidence to prove the truth of the matter asserted in the statement.” A “‘statement’ means a person’s oral assertion,

written assertion, or non-verbal conduct, if the person intended it as an assertion.” Pa.R.E. 801(a). A “‘declarant’ means the person who made the statement.” Pa.R.E. 801(b). “Hearsay is not admissible except as provided by the [Pennsylvania Rules of Evidence], by other rules prescribed by the Pennsylvania Supreme Court, or by statute.” Pa.R.E. 802.

Pa.R.E. 804(b)(1) provides for an exception to the hearsay rule for former testimony where the declarant is unavailable as a witness. See also 42 Pa.C.S.A. § 5917. Pa.R.E. 804(a) sets forth criteria for being unavailable. A declarant is considered to be unavailable as a witness under a number of circumstances including where the declarant cannot be present or testify at the trial or hearing because of death. Pa.R.E. 804(a)(4).

Pa.R.E. 804(b)(1) sets forth requirements for using the former testimony exception where a declarant is unavailable as a witness as follows:

(b) The Exceptions. The following are not excluded by the rule against hearsay if the declarant is unavailable as a witness:

(1) Former Testimony. Testimony that: (A) was given as a witness at a trial, hearing, or lawful deposition, whether given during the current proceeding or a different one; and (B) is now offered against a party who had – or, in a civil case, whose predecessor in interest had – an opportunity and similar motive to develop it by direct, cross-, or redirect examination.

Where a defendant has an opportunity at a preliminary hearing to cross-examine a witness, and elects not to do so, the testimony of the witness will nevertheless be admissible at trial where the

19

Page 24: FEBRUARY 2020 PENNSYLVANIA BAR EXAMINATION · PENNSYLVANIA BAR EXAMINATION Essay Questions and Examiners’ Analyses and Performance Test Pennsylvania Board of Law Examiners 601 Commonwealth

witness later becomes unavailable due to death. See Commonwealth v. Thompson, 648 A.2d. 315, 322 (Pa. 1994) (overruled in part on other grounds).

As applied here, the facts indicate that at Mike’s preliminary hearing, where Mike was

present and represented by counsel, Scott testified that he told Mike that the bike rack was stolen when he gave it to him. Scott’s oral assertion at the preliminary hearing would be deemed to be

a statement, and Scott would be deemed to be the declarant as he was the one who made the statement. Scott’s statement would be hearsay because it is being used for the truth of the matter asserted, i.e. that Scott told Mike the bike was stolen.

Although Scott’s statement would be deemed to be hearsay, he would be considered to be

an unavailable witness due to his death. Since Scott’s statement was made at a preliminary

hearing on the charge against Mike of receiving stolen property, the same charge being litigated at trial, Scott’s preliminary hearing testimony would likely meet the first requirement of the former testimony exception set forth at Pa.R.E. 804(b)(1)(A) above, i.e. that the witness gave testimony at a hearing. The DA would also have to show that Mike’s attorney had an

opportunity and similar motive to develop this testimony on cross-examination under the Pa.R.E. 804(b)(1)(B) cited above. The facts indicate that Mike was present and represented by counsel at the preliminary hearing and that his attorney opted not to cross-examine Scott, although he clearly had the opportunity to do so. Since Mike’s attorney had the opportunity to engage in cross-examination, the requirements of Pa.R.E. 804(b)(1)(B) will have been met if there is evidence that Mike’s attorney had a similar motive to develop the testimony at the preliminary

hearing as at trial. In sum, the court will likely allow Scott’s preliminary hearing testimony to be used

against Mike at his trial under the former testimony exception to the hearsay rule.

4. Neither the Speedster X 20-speed bike nor the Dome bike helmet will be considered

to be marital property subject to equitable distribution in the divorce action

between Sally and Mike.

23 Pa. C.S.A. § 3501(a) provides in pertinent part as follows:

General rule. As used in this chapter, “marital property” means all

property acquired by either party during the marriage and the increase in value of any nonmarital property acquired pursuant to paragraphs (1) and (3) as measured and determined under subsection (a.1). However, marital property does not include:

(1) Property acquired prior to marriage or property acquired in exchange for property acquired prior to the marriage.

. . . (3) Property acquired by gift, except between spouses, bequest, devise or descent or property acquired in exchange for such property. . . .

20

Page 25: FEBRUARY 2020 PENNSYLVANIA BAR EXAMINATION · PENNSYLVANIA BAR EXAMINATION Essay Questions and Examiners’ Analyses and Performance Test Pennsylvania Board of Law Examiners 601 Commonwealth

As applied here, the facts indicate that Mike purchased the Rocket 6 bicycle for $8,000 in April 2017 which was prior to Sally and Mike’s marriage in July 2019. A week after their marriage, Mike sold the Rocket 6 bicycle for $5,000 and purchased a Speedster X for the same amount, i.e. $5,000. Since Mike purchased the Rocket 6 prior to marriage, it would be classified as nonmarital property under 23 Pa. C.S.A. § 3501(a). When Mike sold the Rocket 6 for $5,000, he used the proceeds of the sale of that nonmarital asset to purchase the Speedster X for the same amount. This exchange of property would qualify under the exception set forth in 23 Pa. C.S.A. § 3501(a)(1) and the Speedster X would be deemed to be nonmarital property.

After the marriage, Mike also received a new Dome bike helmet as a gift from his mother

valued at $950. The Dome bike helmet would be deemed to be a gift and would be deemed to be nonmarital property pursuant to 23 Pa. C.S.A § 3501(a)(3) cited above.

In sum, neither the Speedster X bike nor the Dome bike helmet would be considered to be marital property.

21

Page 26: FEBRUARY 2020 PENNSYLVANIA BAR EXAMINATION · PENNSYLVANIA BAR EXAMINATION Essay Questions and Examiners’ Analyses and Performance Test Pennsylvania Board of Law Examiners 601 Commonwealth

Question No. 3: Grading Guidelines

1. Criminal Law.

Comments: The applicants should discuss the elements of receiving stolen property and conclude that the facts likely support that charge.

4 Points

2. Criminal Procedure.

Comments: The applicants should identify and discuss the principal of abandonment or reasonable expectation of privacy and conclude that Officer Lee’s seizure of the bike rack was

likely constitutional and can be used as evidence against Mike at his trial on the charge of receiving stolen property.

4 Points

3. Evidence.

Comments: The applicants should define and discuss hearsay and the former testimony exception to the hearsay rule and conclude that the court will likely admit Scott’s preliminary

hearing testimony at Mike’s trial.

7 Points 4. Family Law.

Comments: The applicants should define and discuss marital property, the exception for property acquired in exchange for property acquired prior to marriage and the gift exception and conclude that neither the Speedster X bike nor the Dome bike helmet would be considered to be marital property.

5 Points

22

Page 27: FEBRUARY 2020 PENNSYLVANIA BAR EXAMINATION · PENNSYLVANIA BAR EXAMINATION Essay Questions and Examiners’ Analyses and Performance Test Pennsylvania Board of Law Examiners 601 Commonwealth

Question No. 4

By statute, State A allows an individual to be involuntarily committed to a psychiatric

hospital if (1) State A establishes that an individual is dangerous or (2) a court finds a criminal

defendant mentally incompetent to stand trial. (MH Law) The MH Law does not require an

immediate review of the continuation of an involuntary commitment if State A drops criminal

charges against an individual committed solely on the basis of incompetence to stand trial.

However, the MH Law does require a review of all involuntary commitments every 6 months (6-

Month Review).

In 2017, State A charged Kate with theft and remanded her to county jail to await trial

because she could not post bail. While incarcerated, Kate became so depressed that she could

not eat, sleep, bathe, or speak. After finding Kate incompetent to stand trial, a court ordered her

involuntary commitment to a state-owned psychiatric facility (Hospital). No evidence was

presented that Kate was dangerous. Two weeks after her involuntary commitment, State A

dropped the criminal charges against Kate, but Kate spent another 5 months in the Hospital until

she was released following a 6-Month Review where State A never argued she was dangerous.

In November 2018, Kate got a job at Big Corp (Corp), a company with 5,000 employees.

Kate’s education and experience met all of the job criteria. When she accepted the job, Kate told

her supervisor that she has major depressive disorder (MDD), which she manages with

medication and therapy. Without treatment, Kate struggles daily to eat, sleep, bathe, and

communicate because of MDD but, with treatment, MDD does not affect Kate’s daily life.

Kate learned her job quickly, received positive feedback from customers, and completed

all assignments satisfactorily. At the end of a 3-month probationary period, however, Kate’s

supervisor, who has complete control over the terms of Kate’s employment, told Kate that Corp

was not offering her a permanent position because Corp found someone “who is more energetic

23

Page 28: FEBRUARY 2020 PENNSYLVANIA BAR EXAMINATION · PENNSYLVANIA BAR EXAMINATION Essay Questions and Examiners’ Analyses and Performance Test Pennsylvania Board of Law Examiners 601 Commonwealth

and better equipped to handle the stress.” Kate’s supervisor showed her the following comments

that he had put in her personnel file: “11/28 – left ‘sick,’ but Kate seems anxious and

overwhelmed; 12/18 – 45 min. late because of “car trouble,” but probably couldn’t get out of

bed; 12/29 – complained a co-worker yelled at her . . . Kate’s probably being irrational.”

Another probationary employee who does not have any mental or physical health problems was

offered a permanent position.

Kate has sued State A under the United States Constitution. Kate has also sued Corp for

allegedly violating the Americans with Disabilities Act (ADA) by failing to offer her a

permanent job. Both lawsuits were timely filed in an appropriate federal court against proper

parties and all procedural, jurisdictional, and administrative prerequisites are satisfied.

While investigating Kate’s ADA claim, her attorney (JD) took extensive notes (Notes) on

an interview of Bob, Kate’s former co-worker at Corp. The next day, Bob moved to a remote

part of South America, and it would take significant time and expense to locate and contact him.

Bob is the only Corp employee who worked with Kate the day she left sick and was the only

person who saw the dispute with the co-worker that she complained about on December 29. The

Notes include Bob’s description of these events. The Notes also include JD’s thoughts and

opinions about Bob’s credibility and how Bob’s observations affect Kate’s ADA claim. The

Notes do not have any information provided by Kate to JD.

1. If Kate brings a federal procedural due process claim against the appropriate State A official based on her continued involuntary commitment for 5 months after State A dropped the theft charge, how will the court analyze her claim and with what likelihood of success?

2. Will Kate be able to establish a prima facie case of discrimination under the

ADA? 3. During discovery in the ADA case, Corp requests that JD produce the Notes.

What grounds should JD raise to object to disclosure of the Notes, and how will the court likely rule?

24

Page 29: FEBRUARY 2020 PENNSYLVANIA BAR EXAMINATION · PENNSYLVANIA BAR EXAMINATION Essay Questions and Examiners’ Analyses and Performance Test Pennsylvania Board of Law Examiners 601 Commonwealth

Question No. 4: Examiner’s Analysis

1. Kate will likely be successful in asserting a Fourteenth Amendment Procedural Due

Process Claim

The Fourteenth Amendment of the United States Constitution provides that no state shall “deprive any person of life, liberty, or property, without due process of law.” U.S. Const.

Amend. XIV, § 1. “Procedural due process imposes constraints on governmental decisions which deprive individuals of ‘liberty’ or ‘property’ interests within the meaning of the Due Process Clause of the Fifth or Fourteenth Amendment.” Mathews v. Eldridge, 424 U.S. 319, 332 (1976).

Here, Kate must first establish that her claim involves a constitutionally protected interest that triggers the right to procedural due process, which she can easily do. “[C]ivil commitment

for any purpose constitutes a significant deprivation of liberty that requires due process protection.” Addington v. Texas, 441 U.S. 418, 425 (1979) (citing Jackson v. Indiana, 406 U.S. 715 (1972); Humphry v. Cady, 405 U.S. 504 (1972); In re Gault, 387 U.S. 1 (1967); Specht v.

Patterson, 386 U.S. 605 (1967)).

Once a constitutionally protected interest is identified, the court will next determine the “specific dictates of due process,” which requires consideration of three factors:

First, the private interest that will be affected by the official action; second, the risk of an erroneous deprivation of such interest through the procedures used, and the probable value, if any, of additional or substitute procedural safeguards; and, finally, the Government's interest, including the function involved and the fiscal and administrative burdens that the additional or substitute procedural requirement would entail.

Mathews, 424 U.S. at 335.

Applying the first two Mathews factors makes it clear that Kate was entitled to additional due process once the criminal charges against her were dropped. As set forth above, Kate’s

interest affected is “a significant deprivation of liberty.” Addington, 441 U.S. at 425 (emphasis added). Further, as evidenced by what happened to Kate, State A’s procedure of waiting until

the next 6-month review hearing to review an involuntary commitment based solely on incapacity to stand trial carries substantial risk of an “erroneous deprivation” of this liberty interest. Mathews, 424 U.S. at 335. Once the criminal charges against Kate were dropped, given the fact that she was not dangerous, there was no constitutional basis on which State A could continue to confine her to Hospital, thus making her continued commitment improper. O’Connor v. Donaldson, 422 U.S. 563, 576 (1975). (“[A] State cannot constitutionally confine [a mentally ill] nondangerous individual who is capable of surviving safely in freedom by himself or with the help of willing and responsible family members and friends.”). Further, there

is probable, if not definitive, value in State A implementing alternative procedures or safeguards to protect against such erroneous deprivation. State A could, as a matter of law and practice, require the immediate release or commencement of civil commitment proceedings upon dropping criminal charges against an individual subject to involuntary commitment on the basis

25

Page 30: FEBRUARY 2020 PENNSYLVANIA BAR EXAMINATION · PENNSYLVANIA BAR EXAMINATION Essay Questions and Examiners’ Analyses and Performance Test Pennsylvania Board of Law Examiners 601 Commonwealth

of incompetence to stand trial. Mathews, 424 U.S. at 335; see also Jackson v. Indiana, 406 U.S. 715 (1972).1

Finally, the court must consider “the Government's interest, including the function

involved and the fiscal and administrative burdens that the additional or substitute procedural requirement would entail.” Mathews, 424 U.S. at 335. Here, the government may argue that the purpose of the MH Law is to protect society from dangerous individuals. However, the facts specifically state that there is no evidence that Kate is dangerous. In fact, it is difficult to conceive of a State A interest that could outweigh Kate’s liberty interest. Most likely, releasing Kate immediately after dropping the charges would actually reduce the “fiscal and administrative burden” of continued

commitment. Mathews, 424 U.S. at 335. Alternatively, State A will have to make the required showing of dangerousness at some point if it wants to continue the commitment after charges are dropped. Doing so sooner, rather than later, does not seem likely to significantly burden State A, as the costs would be the same no matter when a hearing is held.

Given the strong personal interest at stake, the significant risk of an erroneous deprivation

of liberty, and the minimal government interest in delay, the Fourteenth Amendment likely requires that State A provide notice and a hearing soon after dropping charges against involuntarily committed criminal defendants to determine whether there is a constitutional basis to continue their commitment. Matthews, 424 U.S. at 333 (“The fundamental requirement of due

process is the opportunity to be heard at a meaningful time and in a meaningful matter.”)

(citations and quotation omitted); Mullane v. Central Hanover Bank & Trust, 339 U.S. 306, 314 (1950) (“An elementary and fundamental requirement of due process in any proceeding which is to be accorded finality is notice reasonably calculated, under all circumstances, to apprise interested parties of the pendency of the action and afford them an opportunity to present their objections.”). Here, no such notice or hearing was provided for over five months, resulting in

Kate’s continued, unconstitutional deprivation of liberty. Kate will likely be successful in her claim that State A’s process of waiting for a 6-month

hearing to review the involuntary commitment of non-dangerous criminal defendants violates the procedural due process requirements of the Fourteenth Amendment.

2. Kate will be able to establish a prima facie case of disability discrimination

The Americans with Disabilities Act (ADA) provides as follows: “No covered entity shall discriminate against a qualified individual on the basis of disability in regard to job 1 In Jackson, the Supreme Court held that if it was determined that a criminal defendant subject to a mental health commitment based on incapacity to stand trial would not become competent in the near future, “then the State must

either institute the customary civil commitment proceeding that would be required to commit indefinitely any other citizen, or release the defendant.” 406 U.S. at 738. (“We hold, consequently, that a person charged by a State with a criminal offense who is committed solely on account of his incapacity to proceed to trial cannot be held more than the reasonable period of time necessary to determine whether there is a substantial probability that he will attain that capacity in the foreseeable future.”).

26

Page 31: FEBRUARY 2020 PENNSYLVANIA BAR EXAMINATION · PENNSYLVANIA BAR EXAMINATION Essay Questions and Examiners’ Analyses and Performance Test Pennsylvania Board of Law Examiners 601 Commonwealth

application procedures, the hiring, advancement, or discharge of employees, employee compensation, job training, and other terms, conditions, and privileges of employment.” 42 U.S.C. § 12112(a) (emphasis added).

A “covered entity” includes any employer that employs at least 15 employees for each working day through 20 or more calendar weeks in the current or preceding calendar year. 42 U.S.C. § 12111(2) & 5(A). Here, Corp has over 5,000 employees, making it a “covered entity”

subject to the provisions of the ADA.

Kate must establish the following to state a prima facie case of discrimination under the ADA: (1) she is disabled within the meaning of the ADA; (2) she is otherwise qualified to perform the essential functions of the job, with or without reasonable accommodations; and (3) she has suffered an adverse employment decision as a result of discrimination. Williams v.

Philadelphia Housing Auth. Police Dep’t, 380 F.3d 751, 761 (3d Cir. 2004) (citations omitted) (superseded by statute on other grounds as stated in Robinson v. First State Cmty. Action Agency, 920 F.3d 182, 184 n. 1-3 (3d Cir. 2019)); see also Giordanno v. City of New York, 274 F.3d 740, 747 (2d Cir. 2001) (including within the elements of a prima facie case of disability discrimination that the “employer is subject to the ADA.”).

The ADA defines “disability” as follows:

(A) a physical or mental impairment that substantially limits one or more major life activities . . . ;

(B) a record of such an impairment; or

(C) being regarded as having such an impairment . . . .

42 U.S.C. § 12102(1).

The ADA includes all of the following as examples of “major life activities”: “caring for oneself, performing manual tasks, seeing, hearing, eating, sleeping, walking, standing, lifting, bending, speaking, breathing, learning, reading, concentrating, thinking, communicating, and working.” 42 U.S.C. § 12102(1).

Kate should have no problem showing that she is disabled within the meaning of the ADA by virtue of her history and current diagnosis of MDD. MDD is a mental impairment that, untreated, has and will substantially limit her ability to eat, sleep, care for herself, and communicate. Williams, 380 F.3d at 765 (referring to plaintiff’s major depressive disorder as

“mental impairment”); 29 C.F.R. § 1630.2(j)(3) (stating that major depressive disorder “in

virtually all cases” substantially limit brain function and will “result in a determination of

coverage” under the ADA as both an actual disability or having a record of a disability). Kate has both a current diagnosis and a record (from her time at the Hospital and ongoing treatment) of MDD.

It is possible Corp would argue that Kate is not substantially limited in any major life activity because she is able to control the symptoms of her MDD with medication and therapy so well that she is able to eat, sleep, and adequately care for herself. However, the “substantially

27

Page 32: FEBRUARY 2020 PENNSYLVANIA BAR EXAMINATION · PENNSYLVANIA BAR EXAMINATION Essay Questions and Examiners’ Analyses and Performance Test Pennsylvania Board of Law Examiners 601 Commonwealth

limits” requirement must be made “without regard to the ameliorative effects of mitigating

measures,” including medication and “learned behavioral or adaptive neurological

modifications.” 42 U.S.C. § 12102(4)(E).2

Having established that she has a “disability” within the meaning of the ADA, Kate must

next show that she is “otherwise qualified to perform the essential functions of the job . . . .”

Williams, 380 F.3d at 761 (internal quotations and citations omitted). A qualified individual “satisfies the requisite skill, experience, education and other job-related requirements of the employment position . . . and, with or without reasonable accommodation, can perform the essential functions of the position.” 29 C.F.R. § 1630.2(m). While the facts do not contain specifics about Kate’s job at Corp, they do say that her education and experience met all of the job requirements, that she learned the job quickly, received positive feedback, and completed all assigned work. Thus, Kate is qualified to perform the essential functions of the job.

Finally, Kate must show that she suffered an adverse employment action “as a result of discrimination.” Williams, 380 F.3d at 761 (internal quotations and citations omitted). Here, the circumstances under which Corp failed to offer Kate permanent employment certainly can give rise to an inference of discrimination. Notably, Corp hired a non-disabled person to fill the position that Kate wanted. Moreover, Kate’s personnel file and her supervisor’s comments when

he fired her suggest that he viewed her disability as problematic, even in the absence of any evidence that MDD interfered with her work performance. Kate’s supervisor repeatedly disregarded Kate’s explanation of workplace events, and instead seemed to attribute mental illness to such events. Instead of Kate being sick, having car trouble, or being unfairly yelled at by a co-worker, her supervisor assumed she was anxious, unable to get out of bed, or irrational. He also told her when Corp did not offer her permanent employment that he thought the job was too stressful for her, even though she had received positive feedback and completed all assignments. Given that he knew of Kate’s MDD, it is possible to infer that the decision not to hire Kate permanently was due to discrimination based on her disability.

In sum, Kate will likely be able to establish a prima facie case of disability discrimination under the ADA.

2 Alternatively, Kate could easily establish both “a record” of a disability by the fact of her diagnoses and hospitalization; and “being regarded as” disabled, as evidenced by her supervisor’s subjective interpretations of events noted in her personnel file. 42 U.S.C. § 12102(1)(B)-(C). Where a plaintiff is not challenging the failure of her employer to make reasonable accommodations, it is not necessary to prove the existence of an actual or a record of a disability. 29 C.F.R. § 1630.2(g)(3). Further, proving that an individual is “regarded as” having a disability under (C) is less burdensome for an ADA plaintiff because there is no need to show that the alleged disability “substantially limits one or more major life activities.” Alexander v. Washington Metro Area Trans., 826 F.3d 544, 547-48 (D.C. Cir. 2016) quoting 42 U.S.C. § 12102(3) (“An individual meets the requirement of ‘being regarded as

having such an impairment’ if the individual establishes that he or she has been subject to an action prohibited under

this chapter because of an actual or perceived physical or mental impairment whether or not the impairment limits or is perceived to limit a major life activity.”). Thus, it is often most prudent to proceed under the “regarded as”

definition of disability within the ADA where an employer’s failure to make a reasonable accommodation is not at

issue. However, it is up to an individual plaintiff whether to “proceed under the ‘actual disability’ and/or ‘record

of’” prong regardless of whether the individual is challenging a covered entity’s failure to make reasonable accommodations or requires a reasonable accommodation.” Id. quoting 29 C.F.R. § 1630(g)(3).

28

Page 33: FEBRUARY 2020 PENNSYLVANIA BAR EXAMINATION · PENNSYLVANIA BAR EXAMINATION Essay Questions and Examiners’ Analyses and Performance Test Pennsylvania Board of Law Examiners 601 Commonwealth

3. JD should object to producing the Notes because they are his work product, but the

court may require disclosure of the Notes in redacted form.

Federal Rule of Civil Procedure Rule 26(b) provides, in relevant part:

Parties may obtain discovery regarding any nonprivileged matter that is relevant to any party’s claim or defense . . . . Information within this scope of discovery need not be admissible in evidence to be discoverable.

Fed. R. Civ. P. 26(b)(1).

The content of the Notes is certainly “relevant to [Corp’s] defense” to Kate’s ADA claim because they contain the observations of Kate’s co-worker to the very events that Kate’s

supervisor identified as the reason Kate was not offered a full-time job. However, the inquiry does not end with relevance. One limitation on the broad scope of discovery set forth in Rule 26(b)(1) is the discovery of attorney work product. Fed. R. Civ. P. 26(b)(3)(A). “Ordinarily, a party may not discover documents and tangible things that are prepared in anticipation of litigation or for trial by or for another party or its [attorney].” Id. However, such work product may be discoverable if the party seeking disclosure can show that the requested information falls within the scope of discovery set forth in Fed. R. Civ. P. 26(b)(1), and that the party seeking discovery “has substantial need for the materials to prepare its case and cannot, without undue

hardship, obtain their substantial equivalent by other means.” Fed. R. Civ. P. 26(b)(3)(A)(i)-(ii); see also Hickman v. Taylor, 329 U.S. 495, 511-12 (1947). If a court orders the disclosure of work product, the court “must protect against disclosure of the mental impressions, conclusions, opinions, or legal theories of a party’s attorney or other representative concerning the litigation.” Fed. R. Civ. P. 26(b)(3)(B) (emphasis added). “At its core, the work-product doctrine shelters the mental processes of the attorney, providing a privileged area within which he can analyze and prepare his client’s case.” U.S. v. Nobles, 422 U.S. 225, 238 (1975).

Here, the Notes fall within the purview of Rule 26(b)(3)(A). They were prepared by JD, Kate’s attorney in the ADA case, in anticipation of litigation and for trial. Thus, JD should object to disclosure of the Notes under the general prohibition on discovery of work product under Rule 26(b)(3)(A).

Notwithstanding this general prohibition on the discovery of work product, the court may

still order JD to produce the Notes if Corp can persuade the court that they satisfy the exception contained in Rule 26(b)(3)(A). Certainly, the Notes would fall within the general scope of discovery in that they are relevant to Kate’s ADA claim and Corp’s defense of the case. Fed. R. Civ. P. 26(b)(3)(A)(i). When Kate’s supervisor told Kate that Corp would not be offering

permanent employment, he specifically referenced three incidents in her personnel file. The Notes contain Bob’s observations regarding two of those incidents – the day Kate left early and the day there was a dispute with another co-worker. The fact that the information contained in the Notes is likely inadmissible hearsay does not impact their general discoverability under Rule 26(b).

Because of the relevance of the Notes to Corp’s defense, combined with the fact that Bob

was the only person who worked with Kate the day she left sick and the day of the dispute

29

Page 34: FEBRUARY 2020 PENNSYLVANIA BAR EXAMINATION · PENNSYLVANIA BAR EXAMINATION Essay Questions and Examiners’ Analyses and Performance Test Pennsylvania Board of Law Examiners 601 Commonwealth

between Kate and her co-worker, Corp can likely also show that it has a substantial need to obtain the Notes in order to prepare its case. Fed. R. Civ. P. 26(b)(3)(A)(ii). Corp deserves to know what Bob told JD and whether it is consistent (or inconsistent) with the notations in Kate’s personnel file in order to adequately defend its case.

Finally, because Bob is not available for interview or deposition due to residing in a

remote geographic location and finding him would require substantial resources, a court may find that it would cause undue hardship, if not be entirely impossible, to obtain the “substantial

equivalent” of the information contained in the Notes in any way other than through discovery. Fed. R. Civ. P. 26(b)(3)(A)(ii).

In sum, JD should object to production of the Notes based on the work product doctrine

set forth in Federal Rule 26(b)(3). However, a court may still require JD to produce the Notes with his mental impressions and opinions redacted because Corp can show that the Notes are relevant, that it has substantial need for them, and that it cannot obtain the information contained in the Notes by other means without undue hardship.

30

Page 35: FEBRUARY 2020 PENNSYLVANIA BAR EXAMINATION · PENNSYLVANIA BAR EXAMINATION Essay Questions and Examiners’ Analyses and Performance Test Pennsylvania Board of Law Examiners 601 Commonwealth

Question No. 4: Grading Guidelines

1. Constitutional Law

Comments: Candidates should demonstrate an understanding of the requirements of procedural due process under the Fourteenth Amendment, apply it to a set of facts, and conclude that Kate would likely be successful in her due process challenge. 5 points 2. Employment Law

Comments: Candidates should demonstrate an understanding of the elements of a prima facie case of discrimination under the Americans with Disabilities Act, apply it to a set of facts, and conclude that Kate would be able to establish a prima facie case of discrimination. 8 points

3. Civil Procedure

Comments: Candidates should demonstrate an understanding of the work product doctrine set forth in Rule 26(b)(3) of the Federal Rules of Civil Procedure, identify the exception to the general rule of non-discoverability of work product, and reach a well-reasoned conclusion. 7 points

31

Page 36: FEBRUARY 2020 PENNSYLVANIA BAR EXAMINATION · PENNSYLVANIA BAR EXAMINATION Essay Questions and Examiners’ Analyses and Performance Test Pennsylvania Board of Law Examiners 601 Commonwealth

Question No. 5

Bill provides support services to companies making movies and television shows in Big City,

Pennsylvania. Bill and Allied Studios (“Allied”) entered into negotiations for a contract under which Bill

would provide skilled workers for a movie that Allied was about to start filming in Big City. The

negotiations included (i) whether Bill’s overhead charge of $2,500/day for providing workers to Allied

would be paid through a state movie incentive program and (ii) the wage rates to be paid for the workers.

Bill and Allied finally agreed that Allied would pay Bill’s $2,500 daily overhead charge and that Allied

would pay workers based upon calendar year 2019 wage rates. Bill and Allied then signed a detailed

written “Services Contract” that stated in part, “This writing is intended as the final, complete, and exclusive

expression of the terms of our agreement. All prior agreements and understandings are merged herein.”

Bill, a widower, lived alone in a large Big City house known as Blackacre. Bill had one child, a

daughter named Ann. Bill and Ann had been estranged for many years despite Bill’s frequent but

unsuccessful attempts to reestablish communications. Wishing to downsize, Bill bought a unit in a

condominium building in the heart of downtown Big City within a short walking distance of restaurants and

shops. He then validly sold Blackacre to Chuck. Bill’s deed stated, “I grant and convey Blackacre to Chuck

and GENERALLY warrant the property conveyed.”

Ann owned a country house known as Greenacre in nearby Rustic, Pennsylvania. Tired of rural life,

Ann purchased a unit in the same building in Big City as her father not knowing that Bill lived there. Ann

then entered into a valid written sales agreement to sell Greenacre to Tom for $200,000. The sales

agreement provided for a $20,000 cash deposit at the time of the signing with the balance to be paid at the

time of closing. The sales agreement did not contain a provision regarding loss in the event of any casualty.

One week before the scheduled closing, torrential rainstorms caused landslides and flooding that destroyed

the house on Greenacre.

32

Page 37: FEBRUARY 2020 PENNSYLVANIA BAR EXAMINATION · PENNSYLVANIA BAR EXAMINATION Essay Questions and Examiners’ Analyses and Performance Test Pennsylvania Board of Law Examiners 601 Commonwealth

Bill saw Ann in the lobby of their condominium building. Bill stated, “Ann, I promise to buy you a

Cartier watch if you’ll have lunch with me next Thursday at Mortie’s, which is only 25 feet from this

building’s entrance, and just talk with me.” Ann met Bill for lunch as scheduled. After their lunch, Ann

and Bill got into an argument, and Bill angrily left the restaurant and never purchased the watch.

1. When Tom refused to go forward with the purchase of Greenacre, Ann filed suit seeking the balance of the purchase price. Applying the common law of property, will Ann’s suit be

successful?

2. Ann demanded that Bill fulfill his promise to buy her the Cartier watch. Bill refused Ann’s

demand, saying, “You did nothing to deserve that watch. It was just a short walk and a lunch!” Without discussing promissory estoppel, any equity-based legal doctrines, or the Statute of Frauds, was an enforceable contract created between Bill and Ann?

3. Chuck received a notice from Big City of the existence of an unpaid but valid lien totaling

$10,000 filed against Blackacre for sewer and water line improvements made prior to Bill’s

ownership of the property. Chuck paid $10,000 to satisfy the lien.

(a) Based solely on the language in the deed, will Chuck be successful in recovering the cost to satisfy the lien from Bill? (b) For this part only, assume that Bill’s deed stated, “Bill SPECIALLY warrants the property conveyed.” Based solely on this language, will Chuck be successful in recovering

the cost to satisfy the lien from Bill?

4. When Allied failed to pay Bill for his services, Bill sued Allied for breach of the Services Contract.

(a) At trial, Allied attempted to introduce evidence that the parties orally agreed just prior to the signing of the Services Contract that Bill’s overhead charge would be paid by the state

incentive program. How should the court rule on an objection to the introduction of this evidence made by Bill based upon the parol evidence rule?

(b) For this part only, assume that the written Services Contract contained an error missed by both parties and incorrectly stated that Allied would pay workers based upon the “2020” instead of “2019” calendar wage rates. Because the 2020 wage rates are much higher than the 2019 rates, Allied wants to introduce evidence of this error. Bill also objected to the introduction of this evidence based upon the parol evidence rule. How should the court rule on the objection?

33

Page 38: FEBRUARY 2020 PENNSYLVANIA BAR EXAMINATION · PENNSYLVANIA BAR EXAMINATION Essay Questions and Examiners’ Analyses and Performance Test Pennsylvania Board of Law Examiners 601 Commonwealth

Question No. 5: Examiner’s Analysis

1. Ann’s suit will be successful. She is entitled to specific performance of the sales agreement and

to receive the balance of the purchase price from Tom. Under the doctrine of equitable

conversion, Tom became the equitable owner of Greenacre and bears the risk of loss for

Greenacre’s destruction prior to the closing.

“[I]t is well-established law . . . that when the Agreement of Sale is signed, the purchaser becomes

the equitable or beneficial owner through the doctrine of equitable conversion.” DiDonato v. Reliance Std.

Life Ins. Co., 249 A.2d 327, 329 (Pa.1969). “Until the deed is transferred, a seller retains legal title as trustee for a buyer . . . .” Mut. Beneficial Ins. Co. v. Goschenhoppen Mut. Ins. Co., 572 A.2d 1275, 1277 (Pa. Super. 1990). “The vendor [also] retains . . . a security interest for the payment of the unpaid purchase money.” DiDonato, 249 A.2d at 329, citing Payne v. Clark, 187 A.2d 769 (Pa. 1963).

“It is also the law of Pennsylvania that the purchaser of real estate bears the risk of loss for injury

occurring to the property after execution of the Agreement of Sale but before the settlement.” DiDonato, 249 A.2d at 329 (citations omitted). “When the agreement of sale is silent as to which party assumes the

risk of loss prior to closing, the common law places this risk on the buyer.” Mut. Beneficial Ins. Co., 572 A.2d at 1277. Moreover, equitable conversion requires the buyer to pay the full contract price to the seller whatever the condition of the property. Partrick & Wilkins Co. v. Reliance Ins. Co., 456 A.2d 1348, 1351 (Pa. 1983).

Based upon the doctrine of equitable conversion, Tom is the equitable owner of Greenacre and bears the risk of loss for any injury to the property that occurs after the execution of the agreement of sale but before delivery of the deed at settlement. Therefore, even though the home on Greenacre was destroyed by a landslide and flooding, Ann is legally entitled to specific performance of the agreement and to receive the balance of the unpaid purchase price from Tom. 2. An enforceable contract was created between Bill and Ann. Ann’s actions in traveling the

short distance from her building to have lunch with Bill constitutes consideration for Bill’s

promise to purchase a Cartier watch for Ann.

“It is black letter law that in order to form an enforceable contract, there must be an offer, acceptance, consideration or mutual meeting of the minds.” Jenkins v. Cty. of Schuylkill, 658 A.2d 380, 383 (Pa. Super. 1995) appeal denied, 666 A.2d 1056 (Pa. 1995). In this case, the initial two elements for an enforceable contract are clearly present. Bill made an offer to buy a Cartier watch for Ann if Ann had lunch with Bill and talked to him, and Ann accepted the offer by performing the requested acts. Thus, whether there is an enforceable contract in this situation is dependent upon the presence of consideration.

The modern concept of consideration is composed of two elements: legal value and bargained-for-exchange. JOHN E. MURRAY, JR., MURRAY ON CONTRACTS § 56 (LEXISNEXIS 5th ed. 2011). The legal value element for consideration requires that a benefit must be conferred upon the promisor or a detriment caused to or suffered by the promisee. Hillcrest Foundation, Inc. v. McFeaters, 2 A.2d 775, 778 (Pa. 1938). “A test of good consideration is whether the promisee, at the instance of the promisor, has done, forborne or undertaken to do anything real, or whether he has suffered any detriment, or whether . . . he has done

34

Page 39: FEBRUARY 2020 PENNSYLVANIA BAR EXAMINATION · PENNSYLVANIA BAR EXAMINATION Essay Questions and Examiners’ Analyses and Performance Test Pennsylvania Board of Law Examiners 601 Commonwealth

something that he was not bound to do, or has promised to do some act, or has abstained from doing something.” Presbyterian Bd. of Foreign Missions v. Smith, 58 A. 689, 690 (1904); see also, RESTATEMENT (SECOND) OF CONTRACTS § 71 (3) (AM. LAW INST. 1981). “The terms ‘benefit’ and

‘detriment’ are used in a technical sense in the definition [of consideration], and have no necessary reference to material advantage or disadvantage to the parties.” Stelmack v. Glen Alden Coal Co., 14 A.2d 127, 128 (Pa. 1940).

There may even be a consideration without the accrual of any benefit at all to the promisor. If the promisee has suffered any detriment however slight, or though he has suffered no real detriment, if he has done what he was not otherwise bound to do, in return for the promise he has given a consideration and the court will not ask whether the promisor was benefited.

Hillcrest Foundation, 2 A.2d at 778 (quoting Weigand v. Std. Motor Co., 167 A. 493, 495 (Pa. Super. 1933)).

The bargained-for-exchange element for consideration requires that “the promise must induce the

detriment and the detriment must induce the promise.” MURRAY § 61 [A] (emphasis in original); see also RESTATEMENT (SECOND) OF CONTRACTS § 71(2) (“A performance or return promise is bargained for if it is sought by the promisor in exchange for his promise and is given by the promisee in exchange for that promise.”). “It is not enough, however, that the promisee has suffered a legal detriment at the request of the promisor. The detriment incurred must be the ‘quid pro quo’, or the ‘price’ of the promise, and the

inducement for which it was made.” Stelmack, 14 A.2d at 128–29 (citations omitted).

The bargained-for-exchange element for consideration, however, must be distinguished from the making of a conditional gift. As explained by the Pennsylvania Supreme Court,

If the promisor merely intends to make a gift to the promisee upon the performance of a condition, the promise is gratuitous and the satisfaction of the condition is not consideration for a contract. The distinction between such a conditional gift and a contract is well illustrated in Williston on Contracts [further citation omitted] where it is said: ‘If a benevolent man says to a tramp,-‘If you go around the corner to the clothing shop there, you

may purchase an overcoat on my credit,’ no reasonable person would understand that the

short walk was requested as the consideration for the promise, but that in the event of the tramp going to the shop the promisor would make him a gift.'

Stelmack, 14 A.2d at 129.

Based upon the stated facts, a court likely would conclude that consideration was present to create an enforceable contract between Bill and Ann. The first element for consideration – legal value – is likely present because Ann’s actions, no matter how slight they may seem to have been, constituted a change of position on her part and resulted in the incurring of a detriment.

The second element for consideration - bargained-for-exchange – also is likely present under the stated facts. Ann’s performance (ending her estrangement with her father by walking the short distance

from her building and meeting Bill for lunch) was bargained for because it appears that the performance of these acts was sought by Bill (the promisor) in exchange for his promise to buy the watch and was given by

35

Page 40: FEBRUARY 2020 PENNSYLVANIA BAR EXAMINATION · PENNSYLVANIA BAR EXAMINATION Essay Questions and Examiners’ Analyses and Performance Test Pennsylvania Board of Law Examiners 601 Commonwealth

Ann (the promisee) in exchange for that promise. By forbearing other choices and voluntarily ending her estrangement with Bill as he requested, Ann’s actions likely would be viewed as being induced by and constituting the quid pro quo for Bill’s promise to buy her the watch. Concomitantly, Bill’s promise to buy

the watch for her likely would be viewed as being induced by and constituting the quid pro quo for Ann’s actions. Additionally, there is nothing in the facts to indicate that Bill had made a conditionally gratuitous promise to Ann. To the contrary, Bill and Ann were estranged, and Bill had made frequent but unsuccessful attempts to reestablish communications with his daughter. Thus, Ann’s actions ending her estrangement

with her father ostensibly induced Bill to make his promise to buy the watch. See MURRAY § 61 [B]; 1 E. ALLAN FARNSWORTH, FARNSWORTH ON CONTRACTS § 2.9, n. 8 (LITTLE, BROWN AND CO. 1992). (hypothetical example of father’s promise to buy an emerald ring for his estranged daughter in exchange for meeting him at Tiffany’s meets bargained for exchange requirement).

In short, Ann’s performance of the requested actions in exchange for Bill’s promise to purchase a

Cartier watch is supported by consideration and created an enforceable contract between the parties. 3(a). Chuck will be able to recover the $10,000 that he spent to satisfy Big City’s lien on Blackacre

because he received a general warranty deed from Bill.

“Traditionally, six warranties protect the purchaser against defects in the quality of title.” Juniata

Valley Bank v. Martin Oil Co., 736 A.2d 650, 660 (Pa. Super. 1999). These six warranties are covenants of seisin, right to convey, freedom from encumbrances, warranty, quiet enjoyment and future assurances. Id. at 660-61. By statute, these warranties or covenants of title have been combined into a single covenant of warranty that is put into effect by using such words as “grant and convey” or “grant, bargain, [or] sell” in a

deed. 21 P.S. §§ 4, 8 (2001); see also, 1 RONALD M. FRIEDMAN, LADNER PENNSYLVANIA REAL ESTATE LAW, 16.06 (c) (6th ed. 2013). “There are three variations to the covenant of warranty, and property may be conveyed through a special warranty, general warranty or quitclaim deed." Juniata Valley Bank, 736 A.2d at 661 n.7.

In the situation described in (a), Bill’s deed stated, “I grant and convey Blackacre to Chuck” and

GENERALLY warrant the property conveyed.” By statute, this language is considered to have the effect of

creating a covenant of general warranty. 21 P.S. § 5 (2001). A covenant of general warranty requires the grantor to defend the grantee against the “lawful claims and demands of all persons whomsoever.” Id. Put in other terms, a grantor conveying property by a general warranty deed “covenants to defend the grantee's

title against all mankind, the whole world.” Kramer v. Dunn, 749 A.2d 984, 991 (Pa. Super. 2000) (internal quotation and citation omitted).

When Bill conveyed Blackacre to Chuck by a general warranty deed, he was “obliged to deliver a

deed that is free of liens.” Zurich Gen. Accident & Liab. Ins. Co. v. Klein, 121 A.2d 893, 895 (Pa. Super. 1956). Because he received a general warranty deed from Bill, Chuck will be able to recover the $10,000 that he spent to satisfy Big City’s lien on Blackacre from Bill even though the lien resulted from actions prior to Bill’s ownership of the property. See id. 3(b). Chuck will not be able to recover the $10,000 that he spent to satisfy Big City’s lien on

Blackacre because the deed described in (b) is a special warranty deed .

In the alternative situation described in (b), the language in Bill’s deed stated, “Bill SPECIALLY

36

Page 41: FEBRUARY 2020 PENNSYLVANIA BAR EXAMINATION · PENNSYLVANIA BAR EXAMINATION Essay Questions and Examiners’ Analyses and Performance Test Pennsylvania Board of Law Examiners 601 Commonwealth

warrants the property conveyed.” By statute, this language is considered to have the effect of creating a

covenant of special warranty. 21 P.S. § 6 (2001).

A covenant of special warranty requires the grantor to “warrant and defend the . . . property . . . against the lawful claims or demands of the grantor or grantors, and all persons claiming by . . . through or under him or them.” Id. In other words, the grantor of a deed containing a covenant of special warranty agrees to defend and hold harmless the grantee “against any adverse claimant with a superior interest in the

land claiming through the grantor.” Leh v. Burke, 331 A.2d 755, 762 (Pa. Super. 1974). In such a case, the existence of a tax lien or other encumbrance on a property conveyed by a special warranty deed does not automatically give the grantee a right of recovery. Id. Rather, “[t]his covenant is breached if there is an

existing encumbrance created by the grantor at the time the deed is delivered.” Id. (quoting Litmans v.

O’Donnell, 98 A.2d 462, 464 (Pa. Super. 1953). Thus, to recover against a grantor under a special warranty deed, a party must show that the grantor “caused or allowed a lien or encumbrance to burden the land at the

time of transfer.” Id.; see also Bloshinski v. Falaz, 79 A.2d 798, 799-800 (Pa. Super.1951).

The facts state that the Big City lien filed against Blackacre was for sewer and water line improvements made prior to Bill’s ownership of the property. Because the Big City lien predated Bill’s

ownership and did not arise under or through any action taken by him, Chuck would not be able to recover out-of-pocket expenses incurred in satisfying the tax lien from Bill if the deed that he received from Bill was a special warranty deed. 4(a). The court should sustain Bill’s objection based upon the parol evidence rule to Allied’s

attempt to introduce evidence that the parties orally agreed just prior to the signing of the

Services Contract that Bill’s overhead charge would be paid by the state incentive program.

“[T]he parol evidence rule seeks to preserve the integrity of written agreements by refusing to permit the contracting parties to attempt to alter the import of their contract through the use of contemporaneous oral declarations.” Rose v. Food Fair Stores, Inc., 262 A.2d 851, 853 (Pa. 1970). The Supreme Court has explained Pennsylvania’s version of the parol evidence rule as follows:

Where the parties, without any fraud or mistake, have deliberately put their engagements in writing, the law declares the writing to be not only the best, but the only, evidence of their agreement. All preliminary negotiations, conversations and verbal agreements are merged in and superseded by the subsequent written contract . . . [. U]nless fraud, accident or mistake be averred, the writing constitutes the agreement between the parties, and its terms and agreements cannot be added to nor subtracted from by parol evidence.

Yocca v. Pittsburgh Steelers Sports, Inc., 854 A.2d 425, 436 (Pa. 2004) (quoting Gianni v. Russell & Co., 126 A. 791, 792 (Pa. 1924)).

In deciding whether parol evidence is admissible to vary the terms of a written agreement, a court must perform two tasks. Green Valley Dry Cleaners, Inc. v. Westmoreland Cty. Indus. Dev. Corp., 832 A.2d 1143, 1154-55 (Pa. Cmwlth. 2003), appeal denied, 851 A.2d 143 (Pa. 2004). First, a court must determine whether the writing was intended to be the entire contract between the parties. This is done by looking at the written document. Gianni, 126 A. at 792. “[I]f it appears to be a contract complete within itself, couched in such terms as import a complete legal obligation without any uncertainty as to the object

37

Page 42: FEBRUARY 2020 PENNSYLVANIA BAR EXAMINATION · PENNSYLVANIA BAR EXAMINATION Essay Questions and Examiners’ Analyses and Performance Test Pennsylvania Board of Law Examiners 601 Commonwealth

or the extent of the engagement, it is conclusively presumed that the whole engagement of the parties, and the extent and manner of the undertaking, were reduced to writing.” Id. (internal quotations and citation omitted). “An integration clause which states that a writing is meant to represent the parties’ entire

agreement is also a clear sign that the writing is meant to be just that and thereby expresses all of the parties’

negotiations, conversations, and agreements made prior to its execution.” Yocca, 854 A.2d at 436 (citations omitted).

Second, a court must determine whether the prior oral agreement or statement comes “within the field” embraced by the subsequent written agreement. Green Valley Dry Cleaners, Inc., 832 A.2d at 1155. This is done by comparing the oral terms with the written terms, “and determining whether the parties . . . would naturally and normally include the one in the other if it were made. If they relate to the same subject-matter and are so interrelated that both would be executed at the same time and in the same contract, the scope of the subsidiary agreement must be taken to be covered by the writing.” Gianni, 126 A. at 792.

In this case, a court likely would conclude that Allied’s attempt to introduce evidence that the

parties orally agreed just prior to the signing of the Services Contract that Bill’s overhead charge would be paid by the state incentive program would be precluded by the parol evidence rule. The Services Contract between Bill and Allied is described in the facts as “detailed.” Additionally, the Services Contract

specifically declared that it was “intended as the final, complete, and exclusive expression of the terms of our agreement” and that “[a]ll prior agreements and understandings are merged herein.” Thus, the Service

Contract appears to have been intended to be the entire contract between the parties.

Second, the subject of the prior or contemporaneous agreement that Allied seeks to introduce, the source for the payment of Bill’s compensation, is addressed in the final written agreement between the

parties. Thus, it is reasonable to assume that the source of the payment of Bill’s compensation would

naturally and normally be covered by the subsequent written agreement. Because it “comes within the

field” embraced by the subsequent written agreement, the writing must have been intended to cover the

matter.

In conclusion, the court should sustain Bill’s objection based upon the parol evidence rule because

Allied’s evidence that Bill’s overhead charge would be paid by the state incentive program directly

contradicts the terms of their integrated written contract. 4(b). The court should overrule Bill’s objection to Allied’s attempt to introduce evidence that the

Services Contract contained an error and incorrectly stated that Allied would pay workers

based upon “2020” instead of “2019” calendar year wage rates because the parol evidence rule

does not apply where a term is incorrectly stated in the contract due to mutual mistake.

Although parol evidence generally cannot be introduced to change the terms of a writing which contains an integration clause because such a clause is intended to represent the complete expression of the parties’ agreement, extrinsic evidence is admissible for the

purpose of showing that by reason of mistake, fraud or accident, the written instrument does not express the actual intention of the parties.

Voracek v. Crown Castle USA, Inc., 907 A.2d 1105, 1107 (Pa. Super. 2006) (internal citations omitted).

38

Page 43: FEBRUARY 2020 PENNSYLVANIA BAR EXAMINATION · PENNSYLVANIA BAR EXAMINATION Essay Questions and Examiners’ Analyses and Performance Test Pennsylvania Board of Law Examiners 601 Commonwealth

In the situation described in (b), the written Services Contract did not express the true intention of the parties. As the facts state, Bill and Allied agreed that Allied would pay workers based upon calendar year 2019 wage rates. The parties, however, mistakenly omitted this term from the written Services Contract and incorrectly stated that workers would be paid based upon calendar year 2020 wage rates. “Where . . . a party alleges that a mutual mistake has been made regarding the terms of the written

instrument, the trial court may consider evidence of such mistake to ascertain the true intent of the parties.” Id.

According to Dean Murray, the mistake made here, the parties’ inclusion of the incorrect calendar year for determining the wage rates that Allied would pay, would be a “scrivener’s error,” an error in the expression of the agreement of the parties. In such an instance, “a court must receive the prior evidence of

the parties’ intention, and such evidence will be admissible notwithstanding the parol evidence rule.” MURRAY § 86 [B]; see also Bokoch v. Noon, 215 A.2d 899, 901 (Pa. 1966) (parol evidence is admissible to correct a scrivener’s error).

Because Allied is attempting to introduce evidence that the Services Contract contained an error made by both parties resulting in the Services Contract incorrectly stating that Allied would pay workers based upon “2020” instead of “2019” calendar year wage rates as the parties had agreed, the evidence comes within the mistake exception to the parol evidence rule. Therefore, the court should overrule Bill’s

objection.

39

Page 44: FEBRUARY 2020 PENNSYLVANIA BAR EXAMINATION · PENNSYLVANIA BAR EXAMINATION Essay Questions and Examiners’ Analyses and Performance Test Pennsylvania Board of Law Examiners 601 Commonwealth

Question No. 5 - Grading Guidelines

1. Equitable Conversion and Risk of Loss Comments: Candidates should discuss the doctrine of equitable conversion and its application to which party bears the risk of loss for damage to a property during the period between the execution of a sales agreement and the delivery of the deed. 3 Points 2. Contract Formation and Elements for Consideration Comments: Candidates should recognize that consideration is one of the three essential elements to any enforceable contract. Candidates should discuss each of the elements necessary for consideration - legal value and bargained-for exchange - and analyze whether these elements are present under the stated facts and thereby create an enforceable contract between the parties. 5 Points 3. General Warranty and Special Warranty Deeds

Comments: Candidates should recognize that Pennsylvania law provides that the use of certain language in a deed indicates whether a deed is a deed of general or special warranty. Candidates should discuss the difference between a general warranty deed and special warranty deed and reach a well-stated conclusion concerning what obligation a grantor has to defend grantee’s title depending on the type of deed. 6 Points

4. Parol Evidence Rule.

Comments: Candidates should discuss the requirements of the parol evidence rule and should apply the requirements of the rule in reaching a well-reasoned conclusion concerning whether the rule would bar the admission of extrinsic evidence which would vary the terms of a written agreement. Candidates further should recognize that the exception to the parol evidence rule allowing extrinsic evidence to be introduced to vary a writing meant to be the parties’ entire contract where a party avers that a term was omitted from the contract because of fraud, accident, or mistake. 6 Points

40

Page 45: FEBRUARY 2020 PENNSYLVANIA BAR EXAMINATION · PENNSYLVANIA BAR EXAMINATION Essay Questions and Examiners’ Analyses and Performance Test Pennsylvania Board of Law Examiners 601 Commonwealth

Question No. 6

Pearl, a resident of C County, Pennsylvania, earned her living as the spokesperson and

“face” of the Perfect Coif line of hair care products. Pearl has very dark hair and tended to have

stray hairs along her chin and upper lip (stray hairs) that had to be removed to enhance her

appearance for Perfect Coif photo shoots. Plucking the hairs periodically was painful and time-

consuming, so Pearl decided to try a new home electrolysis product, the D-Laser, designed for

this purpose and recently manufactured and introduced to the market by D-Lovely Beauty

Products, Inc. (D-Lovely), a C County manufacturer. The D-Laser uses a targeted, narrowly

focused laser beam to destroy hair follicles. The darker one’s hair, the more effective the D-

Laser is.

Pearl purchased a new D-Laser for $100 from the D-Lovely website. The D-Laser was

delivered to Pearl in as-designed condition. The product package included a special pair of

shaded glasses that the consumer was instructed to wear while using the D-Laser in order to

avoid any eye injury from possible misdirection of the laser beam. The package, however, did

not include a shield for areas where hair removal was not desired. A hair shield would have cost

$1 per D-Laser. The instructions on the package also did not suggest shielding nearby hair

growth when using the D-Laser on the face to prevent inadvertent hair loss in case of

misdirection or dispersion of the laser beam.

After carefully reading all instructions and other information provided, Pearl used the D-

Laser repeatedly. As she always does, she strictly adhered to the instructions. The D-Laser was

gradually effective in permanently removing the stray hairs. However, Pearl began to notice

thinning of the normal hair growth on her hairline near the targeted areas. She promptly stopped

41

Page 46: FEBRUARY 2020 PENNSYLVANIA BAR EXAMINATION · PENNSYLVANIA BAR EXAMINATION Essay Questions and Examiners’ Analyses and Performance Test Pennsylvania Board of Law Examiners 601 Commonwealth

using the D-Laser, but the thinning proved to be permanent. As a result, Pearl lost her contract

as the Perfect Coif spokesperson.

Pearl believes her permanent, unwanted hair thinning is the result of dispersion of the

laser beam from her use of the D-Laser to remove stray hairs. She has brought a timely civil

action against D-Lovely in the C County, Pennsylvania Court of Common Pleas.

1. Under Pennsylvania law, excluding intentional torts and negligence, what tort theories should Pearl assert against D-Lovely based on her belief that its D-Laser caused her unintended hair loss?

2. For this question only, assume D-Lovely contracted with another C County corporation, Dandy Distributions, Inc. (Dandy), to package and ship its D-Laser to wholesalers and retailers. D-Lovely alleges it provided printed instructions for use of the D-Laser, to be inserted inside the product packages. D-Lovely alleges the separate instructions contained additional, more detailed information than the package itself and provided additional guidance that would have allowed Pearl to avoid unintended hair loss. However, Dandy failed to include the instructions in the D-Laser packaging. As part of the same action as Pearl’s claim, D-Lovely wants to assert a tort claim against Dandy for failing to include the instructions inside the D-Laser packages as D-Lovely required. Under the Pennsylvania Rules of Civil Procedure, what procedural steps, if any, may D-Lovely take to assert its tort claim against Dandy in the same lawsuit as Pearl’s claim?

3. For this question only, assume that D-Lovely denies that its D-Laser causes unintended hair loss. However, in response to discovery requests from Pearl, D-Lovely produces records from the D-Laser’s research and development phase,

which include anecdotal reports of unintended hair loss experienced by test subjects during product testing. The reports were prepared by Witt Winters, a product development employee at D-Lovely. As part of his job, Winters routinely observes test subjects and prepares reports concerning his observations during the product tests. He maintains all such records in his office at D-Lovely.

Pearl would like to introduce the Winters records into evidence at trial to show that some test subjects experienced unwanted hair loss during the D-Laser product trials. She plans to subpoena Winters to testify concerning the records. Under the Pennsylvania Rules of Evidence, if D-Lovely objects that the records are hearsay, what will Pearl need to establish in order to overcome the hearsay objection?

42

Page 47: FEBRUARY 2020 PENNSYLVANIA BAR EXAMINATION · PENNSYLVANIA BAR EXAMINATION Essay Questions and Examiners’ Analyses and Performance Test Pennsylvania Board of Law Examiners 601 Commonwealth

Question No. 6: Examiner’s Analysis

1. Pearl should assert a product liability claim against D-Lovely based on theories of

defective design and failure to warn.

(a) Defective design

(1) One who sells any product in a defective condition unreasonably dangerous to the user or consumer or to his property is subject to liability for physical harm thereby caused to the ultimate user or consumer, or to his property, if (a) The seller is engaged in the business of selling such a

product, and (b) It is expected to and does reach the user or consumer

without substantial change in the condition in which it is sold.

(2) The rule stated in Subsection (1) applies although

(a) the seller has exercised all possible care in the preparation

and sale of his product, and (b) the user or consumer has not bought the product from or

entered into any contractual relation with the seller.

RESTATEMENT (SECOND) OF TORTS, § 402A (AM. LAW INST. 1965). Section 402A of the Restatement (Second) of Torts is the law of Pennsylvania. Webb v. Zern, 220 A.2d 853, 854 (Pa. 1966); see also Tincher v. Omega Flex, Inc., 104 A.3d 328, 335, 399 (Pa. 2014) (declining to adopt the Restatement (Third) of Torts and reaffirming that § 402A of the Restatement (Second) of Torts, as construed in case law, remains the law of Pennsylvania). In order to establish liability, a plaintiff must prove each of the above factors.

In order to prove a product defect, the plaintiff must establish three elements: (1) The product was defective;

(2) The defect was present when the product left the manufacturer; and

(3) The defect caused the plaintiff’s harm.

Tincher, 104 A.3d at 339. There are two alternative tests, the “consumer expectations” test and the “risk-utility”

test, concerning the applicable standard for determining whether a product is defective under § 402A. Under the “consumer expectations” test, “the product is in a defective condition if the

43

Page 48: FEBRUARY 2020 PENNSYLVANIA BAR EXAMINATION · PENNSYLVANIA BAR EXAMINATION Essay Questions and Examiners’ Analyses and Performance Test Pennsylvania Board of Law Examiners 601 Commonwealth

danger is unknowable and unacceptable to the average or ordinary consumer.” Id. at 387. To prevail in a claim under the consumer expectations test, the plaintiff must establish:

(1) The defendant designed the product;

(2) The product failed to perform as safely as an ordinary consumer would expect,

when used in an intended or reasonably foreseeable manner; and (3) The defect caused the plaintiff’s injury.

Lewis v. Coffing Hoist Dev., Duff-Norton Co., 528 A.2d 590, 592-93 (Pa. 1987); Pa. SSJI (Civ) 16.20.

By contrast, the “risk-utility” test involves a cost-benefit analysis. Under the risk-utility

standard, “a product is in a defective condition if a ‘reasonable person’ would conclude that the probability and seriousness of harm caused by the product outweigh the burden or costs of taking precautions.” Tincher, 104 A.3d at 389. To prevail in a claim under the risk-utility test, the plaintiff must establish:

(1) The defendant designed the product;

(2) A reasonable person would conclude the possibility and seriousness of the harm outweighed the burden or cost of making the product safe; and

(3) The defective condition caused the plaintiff’s harm. See Tincher, 104 A.3d at 389.1

1 In Tincher, the Pennsylvania Supreme Court listed seven factors other jurisdictions have considered in

applying the risk-utility test: (1) The usefulness and desirability of the product, i.e., its utility to the user and to the public as a whole. (2) The safety aspects of the product, i.e., the likelihood that it will cause injury, and the probable

seriousness of the injury. (3) The availability of a substitute product which would meet the same need and not be as unsafe. (4) The manufacturer’s ability to eliminate the unsafe character of the product without impairing its

usefulness or making it too expensive to maintain its utility. (5) The user’s ability to avoid danger by the exercise of care in the use of the product (6) The user’s anticipated awareness of the dangers inherent in the product and their availability, because of

general public knowledge of the obvious condition of the product, or of the existence of suitable warnings or instructions.

(7) The feasibility, on the part of the manufacturer, of spreading the loss by setting the price of the product

or carrying liability insurance.

44

Page 49: FEBRUARY 2020 PENNSYLVANIA BAR EXAMINATION · PENNSYLVANIA BAR EXAMINATION Essay Questions and Examiners’ Analyses and Performance Test Pennsylvania Board of Law Examiners 601 Commonwealth

In Tincher, the Pennsylvania Supreme Court examined the policies underlying both standards, as well as § 402A, and concluded that to prove the defect “in Pennsylvania, the cause

of action in strict products liability requires proof, in the alternative, either of the ordinary consumer’s expectations or of the risk-utility of a product.” Id. at 401 (emphasis added). Accordingly, a plaintiff asserting a strict liability claim alleging a defective product may show “either that (1) the danger is unknowable and unacceptable to the average or ordinary consumer, or that (2) a reasonable person would conclude that the probability and seriousness of harm caused by the product outweigh the burden or costs of taking precautions.” Id. at 335.

Here, Pearl has a viable claim that the D-Laser was defective in design under either

standard. The danger of laser beam dispersion is likely to be unknowable and unacceptable to the average consumer. Alternatively, the risk of unwanted hair loss, with the accompanying loss of self-esteem, and in Pearl’s case, loss of livelihood, far outweighs the minor cost of enclosing an appropriate hair shield for use when operating the laser near the hairline. In fact, the absence of a shield for hair not intended to be removed might itself constitute a design defect. See Lewis, 528 A.2d at 593 (“a product is defective when it leaves the supplier’s control lacking any element necessary to make it safe for its intended use”).

(b) Failure to warn of possible laser beam dispersion

A product not otherwise defective in design may nonetheless be rendered defective where it lacks adequate warnings necessary for its safe use. Jacobini v. V. & O. Press Co., 588 A.2d 476, 478 (Pa. 1991). Of course, the plaintiff must still prove causation, i.e., that the failure to warn was the proximate cause of the injury. See id. at 479 (rejecting failure to warn claim where evidentiary record indicated lack of warning was not cause of injury).

Here, Pearl should assert a product liability claim alleging that D-Lovely failed to warn

of the risk of unintended hair loss posed by its D-Laser.

Whether Pearl will succeed in her claim alleging failure to warn will depend on the evidence adduced at trial. The facts applicable to this interrogatory indicate that the D-Laser packaging did not contain a warning. Only Interrogatory 2 assumes D-Lovely provided Dandy with a replacement instruction sheet containing a warning. Based on the facts provided, therefore, it appears Pearl has a viable claim against D-Lovely for failure to warn about the risk of unintended hair loss posed by possible laser dispersion.

Pearl must still prove that had the warning been given, she would not have incurred

damages, i.e., she would have taken appropriate precautions as recommended. That will be a question of fact dependent on credibility of the evidence at trial.2 Because Pearl’s livelihood

Tincher, 104 A.3d at 389-90, However, the Tincher Court also observed that litigation before a jury generally focuses on the balance between the feasibility and cost-effectiveness of an alternate design and the seriousness of the risk of harm. Id.

2 The Pennsylvania Superior Court has recognized a “heeding presumption” that if a warning necessary to make a

product safe was not given, the plaintiff would have followed the warning if given. Coward v. Owens-Corning

Fiberglas Corp., 729 A.2d 614, 621 (Pa. Super. 1999), appeal granted, 743 A.2d 920 (1999). The defendant may rebut the presumption by producing evidence that the plaintiff would not have heeded the warning. If the defendant

45

Page 50: FEBRUARY 2020 PENNSYLVANIA BAR EXAMINATION · PENNSYLVANIA BAR EXAMINATION Essay Questions and Examiners’ Analyses and Performance Test Pennsylvania Board of Law Examiners 601 Commonwealth

depended on maintaining her hair in good condition and the facts state she always adheres

strictly to product instructions, a jury could conclude she would have followed any precautions

suggested in the D-Laser instructions had she known of the possible unintended hair loss.

2. D-Lovely may join Dandy as an additional defendant by filing a praecipe for a writ

or a complaint against Dandy.

As a general rule, Pa.R.C.P. No. 1033(a) allows any party to add another party to a

pending civil action at any time by leave of court or consent of the other party. However, this

broad provision is limited by Rule 2252 as applied to joinder of additional defendants:

(a) Except as provided by Rule 1706.1 [relating to joinder of additional

defendants in class actions], any party may join as an additional defendant

any person not a party to the action who may be

(1) solely liable on the underlying cause of action against the joining party, or

(2) liable to or with the joining party on any cause of action arising out of the

transaction or occurrence or series of transactions or occurrences upon which the

underlying cause of action against the joining party is based.

Pa.R.C.P. No. 2252(a). See Stokes v. Loyal Order of Moose Lodge No. 696, 466 A.2d 1341,

1345 (Pa. 1983) (holding it was error to permit joinder where complaints did not arise out of the

same transaction or occurrence); 202 Island Car Wash, L.P. v. Monridge Constr., Inc., 913 A.2d

922, 927 (Pa. Super. 2006) (stating “the key inquiry is whether the additional defendant’s

liability is related to the plaintiff’s claim against the original defendant.”).

Joinder of an additional defendant is accomplished by filing a praecipe for a writ of

joinder, or by filing a complaint against the additional defendant. Pa.R.C.P. 2252(b). If the

joinder is by writ, a complaint must be filed within 20 days thereafter. Id. A party may join an

additional defendant as of right during the first 60 days after service of the initial pleading on the

original defendant, or by the deadline for filing the joining party’s answer to the complaint,

whichever is later. Pa.R.C.P. No. 2253(a). Joinder thereafter must be by leave of court or

consent of all parties. Pa.R.C.P. No. 2253(a).

Here, D-Lovely contends that Dandy failed to include separate printed product

instructions and information concerning the D-Laser that would have allowed Pearl to avoid

unintended hair loss. The facts set forth in Interrogatory 2 connect D-Lovely’s averments

against Dandy with Pearl’s claim based on failure to warn. If the evidence proves D-Lovely

provided the instructions to Dandy and those instructions constituted a sufficient warning to

consumers, Dandy might be liable to indemnify D-Lovely for any liability to Pearl.

produces such evidence, the burden shifts back to the plaintiff to produce evidence showing she would have acted to

avoid the harm if the warning had been given. Id. at 622. The Pennsylvania Supreme Court has not ruled on

whether Pennsylvania law includes such a presumption. Moreover, it is not clear that the presumption applies

outside the context of asbestos cases.

46

Page 51: FEBRUARY 2020 PENNSYLVANIA BAR EXAMINATION · PENNSYLVANIA BAR EXAMINATION Essay Questions and Examiners’ Analyses and Performance Test Pennsylvania Board of Law Examiners 601 Commonwealth

both D-Lovely’s conduct and Dandy’s were substantial factors in causing Pearl’s damages, Dandy might be jointly or severally liable along with D-Lovely on Pearl’s failure-to-warn claim. D-Lovely may join Dandy as a party in Pearl’s lawsuit as an additional defendant on either basis, or both bases, under Rule 2252.

3. Pearl will need to establish the records of test subject reports of unintended hair loss

are admissible under the business records exception to the hearsay rule.

“Hearsay” is defined as “a statement that (1) the declarant does not make while testifying

at the current trial or hearing; and (2) a party offers in evidence to prove the truth of the matter asserted in the statement.” Pa.R.E. 801(c). Under Pa.R.E. 802, hearsay is not admissible in a court proceeding unless it is subject to an exception provided by another rule of evidence or a statute.

Here, the test subject records were created by Witt Winters, but not while testifying at

trial. Pearl wants to offer the records for the truth of the matters asserted, specifically that some test subjects experienced unwanted hair loss during the product trials. Therefore, the records constitute hearsay.

The Pennsylvania Rules of Evidence list a number of exceptions to the general rule

precluding admission of hearsay. Rule 803 lists exceptions that apply regardless of the declarant’s availability as a witness. One of them, commonly known as the business records exception, provides for admissibility of:

(6) Records of a Regularly Conducted Activity. A record (which includes a memorandum, report, or data compilation in any form) of an act, event, or condition if:

(A) the record was made at or near the time by – or from

information transmitted by – someone with knowledge; (B) the record was kept in the course of a regularly conducted

activity of a “business,” which term includes business,

institution, association, profession, occupation, and calling of every kind, whether or not conducted for profit;

(C) making the record was a regular practice of that activity;

(D) all these conditions are shown by the testimony of the

custodian or another qualified witness, or by a certification that complies with Rule 902(11) or (12) or with a statute permitting certification; and

(E) the opponent does not show that the source of information

or other circumstances indicate a lack of trustworthiness.

47

Page 52: FEBRUARY 2020 PENNSYLVANIA BAR EXAMINATION · PENNSYLVANIA BAR EXAMINATION Essay Questions and Examiners’ Analyses and Performance Test Pennsylvania Board of Law Examiners 601 Commonwealth

Pa.R.E. 803(6).3

Applying Rule 803(6) to the facts as stated, Pearl may demonstrate that the test subject reports concerning the D-Laser are business records if she meets the requirement of each subpart of the rule.

First, Pearl must establish that the records were made at or near the time the side effects

occurred, and that they were made by someone with knowledge of the occurrences, or by someone with information supplied by someone with such knowledge. Second, Pearl must establish that D-Lovely kept the test subject reports in the course of its regularly conducted business activities. Third, she must establish that making such records was D-Lovely’s regular practice during such activities. Pearl must establish each of these required facts through either the testimony of a records custodian or other qualified witness from D-Lovely, or by a certification meeting the signing/verification requirements of Pa.R.E. 902(11) and Pa.R.C.P. No. 76. In order to oppose the admission of the record, D-Lovely must be able to show that the source of the records or some other circumstances indicate that the records lack trustworthiness. Assuming that the test subject reports meet the requirements of Rule 803(6), they will be admissible as business records. See Spino v. John S. Tilley Ladder Co., 696 A.2d 1169, 1171, 1175 (Pa. 1997) (manufacturer’s chronological log of all reported consumer problems with its ladder was admissible as a business record).4

3 Rule 902(12) relates to foreign corporations. As the facts indicate both D-Lovely and Dandy are

Pennsylvania companies, that rule is inapplicable. Rule 902(11) provides that business records are self-authenticating if accompanied by a certification complying with Pa.R.C.P. No. 76. Rule 902(11) further requires that to seek a ruling that a record is self-authenticating, the proponent must give reasonable advance written notice to his opponent concerning his intent to offer the record in evidence and must make the record available for inspection so that his opponent has an opportunity to challenge it.

Rule 76 of the Pennsylvania Rules of Civil Procedure is a set of defined terms. Rule 902(11)’s reference to Pa.R.C.P. No. 76 presumably relates to the definition of “verified,” which “when used in reference to a written

statement of fact by the signer, means supported by oath or affirmation or made subject to the penalties of 18 Pa.C.S. § 4904 relating to unsworn falsification to authorities.” Pa. R.C.P. No. 76.

4 An initial reading of the facts might suggest an alternative basis for admission of the records, i.e., an opposing party statement under Pa. R.E. 803(25). This hearsay exception potentially applies where the document is authenticated, the declarant was an employee of the opposing party when the statement in the document was made, and the statement referred to work within the employee’s scope of work. Id. However, these factors alone do not establish admissibility without proof of the employee’s agency and authority to make a statement binding on the employer. See DiFrancesco v. W. Pa. Water Co., 478 A.2d 1295, 1300 (Pa. Super. 1984). Here, the facts do not specifically establish agency and authority on the part of Winters.

48

Page 53: FEBRUARY 2020 PENNSYLVANIA BAR EXAMINATION · PENNSYLVANIA BAR EXAMINATION Essay Questions and Examiners’ Analyses and Performance Test Pennsylvania Board of Law Examiners 601 Commonwealth

Question No. 6: Grading Guidelines

1. Torts – Product Liability Comments: The applicant should set forth the elements of a product liability claim for Pearl’s allegations that the D-Laser was (a) defective as designed, and (b) rendered defective by the absence of warnings of possible unintended hair loss. The applicant should conclude that Pearl should assert a claim on theory (a) and that both the consumer expectation and risk-utility standards potentially apply. The applicant should conclude Pearl also should assert a claim on theory (b) for failure to warn, on the basis that D-Lovely knew or should have known the risk of hair loss associated with use of its D-Laser and that it failed to adequately warn Pearl.

10 points

2. Civil Procedure – Joinder of Additional Defendant Comments: The applicant should recognize that because Dandy may be liable over or liable along with D-Lovely on Pearl’s failure-to-warn claim, D-Lovely should join Dandy as an additional defendant in Pearl’s pending lawsuit. The applicant should therefore set forth and describe the applicable rule governing joinder of an additional defendant as provided in the Pennsylvania Rules of Civil Procedure.

5 points

3. Evidence – Business Records Exception to the Hearsay Rule Comments: The applicant should state the hearsay rule and explain the business records exception to the rule. The applicant should apply the requirements for the business records exception to the facts as given, including authentication of the records by the records custodian.

5 points

49

Page 54: FEBRUARY 2020 PENNSYLVANIA BAR EXAMINATION · PENNSYLVANIA BAR EXAMINATION Essay Questions and Examiners’ Analyses and Performance Test Pennsylvania Board of Law Examiners 601 Commonwealth

PT

Supreme Court of Pennsylvania

Pennsylvania Board of Law Examiners

Pennsylvania Bar Examination

February 24 and 25, 2015

PERFORMANCE TEST

February 24, 2015

Use GRAY covered book for your answer to the Performance Test.

© 2015 Pennsylvania Board of Law Examiners

PLACE BAR CODED APPLICANT LABEL HERE

Question Number 3

on Examplify

50

Page 55: FEBRUARY 2020 PENNSYLVANIA BAR EXAMINATION · PENNSYLVANIA BAR EXAMINATION Essay Questions and Examiners’ Analyses and Performance Test Pennsylvania Board of Law Examiners 601 Commonwealth

Table of Contents FILE

1. Assignment Memorandum ........................................................................................... 1

2. Memorandum Containing Instructions for Drafting Client Opinion Letters ............... 2 3. Plaintiff’s Complaint .................................................................................................... 3

LIBRARY

1. Loughran v. The Phillies and Marlon Byrd, 888 A.2d 872 (Pa.Super. 2005) ............. 9

51

Page 56: FEBRUARY 2020 PENNSYLVANIA BAR EXAMINATION · PENNSYLVANIA BAR EXAMINATION Essay Questions and Examiners’ Analyses and Performance Test Pennsylvania Board of Law Examiners 601 Commonwealth

FILE

52

Page 57: FEBRUARY 2020 PENNSYLVANIA BAR EXAMINATION · PENNSYLVANIA BAR EXAMINATION Essay Questions and Examiners’ Analyses and Performance Test Pennsylvania Board of Law Examiners 601 Commonwealth

Memorandum

TO: Applicant FROM: Sarah Benjamin, Managing Partner RE: The William Penns, Inc.; File # 06032005 DATE: February 25, 2020

Our client, The William Penns, Inc., which owns a professional hockey team (The William Penns), was served yesterday with a complaint concerning injuries that arose out of a recent incident at a hockey game played in their home arena in Big City, William County, Pennsylvania. For the sake of brevity, I will refer to our corporate client, The William Penns, Inc., and The William Penns, collectively as “The

Penns.” However, the team is not a separate entity; rather, it is an asset owned by The William Penns, Inc., and its players are employees of The William Penns, Inc. For your information, the mascot for The Penns is a character dressed in a traditional Quaker outfit (seventeenth century clothing including a vest, jacket, knickers for pants, long socks, and a black hat – as you may imagine William Penn looking when he founded our Commonwealth). The Penns’ management is a sophisticated group and they have asked us to provide them with an analysis of the law as it relates to the no-duty rule in Pennsylvania. The Penns’ management will determine how it will proceed with the litigation based upon our letter. No answer to the complaint has been filed yet, but The Penns’ management has indicated that the facts are consistent with those alleged in the complaint. Your assignment is to draft an opinion letter to the client providing a complete legal analysis of why the no-duty rule might or might not apply, and come to a conclusion indicating whether you believe The Penns may rely upon that rule as a defense to the claims in the complaint. Your opinion letter should conform to our firm’s opinion letter guidelines, which are included in this File. Included in this File is the Plaintiff’s timely-filed complaint and attached exhibit, which properly names our client and was properly served. Included in the attached Library is a case relevant to the no-duty rule. You may cite to both the majority and dissenting opinions of the case for your assignment. In any event, you should cite to the case and the complaint and its Exhibit A. You should only use facts contained in the File along with the attached caselaw for your letter to the client. Do not rely upon your personal knowledge of these issues or on rules, cases, or statutes not included in the Library. Instead you should base your analysis and conclusions only upon the documents provided in the File and the Library.

53

Page 58: FEBRUARY 2020 PENNSYLVANIA BAR EXAMINATION · PENNSYLVANIA BAR EXAMINATION Essay Questions and Examiners’ Analyses and Performance Test Pennsylvania Board of Law Examiners 601 Commonwealth

Memorandum

TO: All Attorneys FROM: Sarah Benjamin, Managing Partner RE: Instructions for Drafting Client Opinion Letters DATE: January 1, 2020

The following instructions for drafting client opinion letters must be followed by all firm attorneys: The document should follow the format of a formal business letter. The letter should be dated and directed to the client(s), but neither our letterhead nor the client’s address is required, as these will be added by your assistant. The subject line should succinctly identify the subject matter(s) of the letter, and the client’s file number. 1. The letter should begin with an appropriate salutation followed by a brief statement of the purpose of the letter. 2. The letter should be divided into sections, one for each question presented, which should be separated by a short heading that reflects the issue being addressed. 3. Each section should include a statement of the question being addressed, a reasoned analysis supporting your conclusion that applies the relevant legal principles to the facts, and a conclusion. If there are facts and/or legal principles relevant to any point or element in your analysis that could be argued to support a different conclusion, identify and discuss those principles/facts. 4. You may qualify your opinion if the outcome is less than certain, but if you do, you should state why you cannot give an unqualified opinion. 5. Legal authorities and documents that are relied upon should be cited. Short, informal citation forms are permissible (Bluebook format is not required). 6. The letter should conclude with an appropriate closing and the signature of the assigning partner. Do not include your name on the letter. 7. The opinion letter should be straightforward and logical, and it should be in language that a layperson can understand.

54

Page 59: FEBRUARY 2020 PENNSYLVANIA BAR EXAMINATION · PENNSYLVANIA BAR EXAMINATION Essay Questions and Examiners’ Analyses and Performance Test Pennsylvania Board of Law Examiners 601 Commonwealth

MARIA FENDICK, Plaintiff : In the Court of Common Pleas

: of William County

v. :

: Civil Action - Law

:

THE WILLIAM PENNS, INC., Defendant : No. 2020 CIV 6031995

COMPLAINT

AND NOW, comes the Plaintiff, Maria Fendick, and in support of her Complaint avers as

follows:

1. Plaintiff, Maria Fendick, is an adult competent individual who resides at 645 Main Street,

Big City, William County, Pennsylvania.

2. Defendant, The William Penns, Inc., is a Pennsylvania corporation that owns The

William Penns professional hockey team; it owns and operates the arena where the hockey team plays

its home games; it has its principal place of business located at 1 Penns Plaza, Big City, William

County, Pennsylvania; and it regularly conducts business in William County, Pennsylvania. Defendant

shall be referred to throughout Plaintiff’s Complaint as The Penns.

3. On October 26, 2020, Plaintiff was a ticket-holder for and attended a regular season

professional hockey game hosted by The Penns at their arena located at 1 Penns Plaza, Big City,

William County, Pennsylvania; the game was played against The Penns’ cross-state rival, the Coal City

Cougars (Cougars).

4. Ms. Fendick’s ticket was for a seat in the front row, immediately behind the protective

glass that is designed to prevent players and objects from flying from the playing surface into the stands.

She paid a premium for her ticket so that she could enjoy the game without having to worry about being

hit by a hockey puck.

5. At approximately 7:45 p.m. on January 3, 2020, mid-way through the first period of the

game, Ms. Fendick stood up to better see the hockey puck in play, and was struck in the back of the head

by an octopus carcass that had been hurled by a fan in an attempt to get the deceased creature onto the

55

Page 60: FEBRUARY 2020 PENNSYLVANIA BAR EXAMINATION · PENNSYLVANIA BAR EXAMINATION Essay Questions and Examiners’ Analyses and Performance Test Pennsylvania Board of Law Examiners 601 Commonwealth

ice. An excerpt of a Wikipedia article concerning the purported rationale for fans throwing objects, such

as an octopus, onto the ice is attached hereto for reference as “Exhibit A.”

6. After striking Ms. Fendick in the head, the octopus then bounced over the protective glass

and landed on the ice.

7. The force of the octopus hitting the back of Ms. Fendick’s head pushed her head forward

into the protective glass, splitting her forehead open near her hairline.

8. Immediately after the sudden impact to the rear of her head that also resulted in the

impact to the front of her head, Ms. Fendick sat down in her seat and held her forehead as it was

bleeding profusely. She tilted her head back to apply pressure to the wound and try to keep the blood

off of her face.

9. As Ms. Fendick tilted her head back, she was struck in the right eye by an object that she

would later come to learn was the same octopus that an unidentified player for The Penns had scooped

up with his hockey stick and thrown back over the protective glass into the stands so that the octopus

would not freeze to the ice and interfere with play.

10. Ms. Fendick was attended to by emergency medical staff on duty at the arena, and they

determined that she needed to be transported as quickly as possible to William Community Medical

Center via ambulance.

11. Ms. Fendick was treated at the William Community Medical Center, where she was

admitted overnight and discharged the following day.

12. Ms. Fendick’s injuries include, but are not limited to, the following:

a. concussion;

b. laceration on her forehead requiring 38 stitches;

c. scratched cornea on her right eye requiring her to wear a temporary eye patch and

administer medicated eye drops;

d. strained neck muscles;

56

Page 61: FEBRUARY 2020 PENNSYLVANIA BAR EXAMINATION · PENNSYLVANIA BAR EXAMINATION Essay Questions and Examiners’ Analyses and Performance Test Pennsylvania Board of Law Examiners 601 Commonwealth

e. permanent scarring; and

f. headaches.

13. In addition to the physical injuries listed above, Ms. Fendick has also suffered financially,

as she is paid by-the-hour and is not paid for any time she is not at work. She has not been able to work

since the date of this incident.

14. As the owner and operator of the arena, The Penns had a duty to safeguard Ms. Fendick

from harm.

15. The Penns’ mascot is a man dressed in a Quaker outfit; it is neither common nor expected

that a fan of The Penns would throw an octopus onto the ice at a hockey game of The Penns.

16. The Cougars’ mascot is a cougar, also known as a mountain lion; it is neither common

nor expected that a Cougar’s fan would throw an octopus onto the ice at a Cougars’ hockey game.

17. Prior to being struck by the octopus, Ms. Fendick had never heard of the practice of

throwing objects, such as an octopus, onto the ice at a hockey game; and Ms. Fendick had never heard of

a hockey player throwing an object into the stands with his hockey stick, except for a hockey puck on an

errant pass or shot on goal.

18. Alternatively, if throwing an object such as an octopus onto the ice was common or

expected, The Penns took no action whatsoever to discourage fans from throwing objects of any sort

onto the ice, and it instituted no safeguards that would protect fans against other fans doing just that.

19. It is believed and averred that the hockey player who threw the octopus back into the

stands was at all times a servant, agent, and/or employee of Defendant acting within the course and

scope of his employment.

COUNT I – NEGLIGENCE

20. Plaintiff incorporates each and every paragraph by reference as if more fully set forth

herein.

57

Page 62: FEBRUARY 2020 PENNSYLVANIA BAR EXAMINATION · PENNSYLVANIA BAR EXAMINATION Essay Questions and Examiners’ Analyses and Performance Test Pennsylvania Board of Law Examiners 601 Commonwealth

21. Defendant had a duty to protect Plaintiff from objects being thrown to and from the ice.

22. Defendant breached its duty by failing to prevent fans from bringing objects into the

arena that could be thrown onto the ice during hockey games at Defendant’s facilities.

23. Defendant breached its duty to Plaintiff by failing to institute safeguards against fans

throwing objects toward the ice at hockey games at Defendant’s facilities.

24. Defendant breached its duty by failing to prevent its players; who are its agents, servants,

and/or employees; from throwing objects into the stands during hockey games at Defendant’s facilities.

25. Defendant’s failure to prevent fans from bringing objects, including an octopus, into the

arena that could be thrown by fans onto the ice during hockey games at Defendant’s facilities caused

Ms. Fendick to become severely injured when she was struck by such an object.

26. Defendant’s failure to institute safeguards against fans throwing objects toward the ice at

hockey games at Defendant’s facilities caused Ms. Fendick to become severely injured upon being

struck by an octopus thrown toward the ice by a fan.

27. Defendant’s failure to prevent its players, who are its agents, servants and/or employees

from throwing objects into the stands during hockey games at Defendant’s facilities caused Ms. Fendick

to become severely injured upon being struck by an octopus thrown into the stands by Defendant’s

player.

Wherefore, Plaintiff respectfully requests that the Court enter judgment in her favor and against

Defendant in an amount in excess of the arbitration limits.

Respectfully Submitted,

/s/ Atticus Mason

Atticus Mason, Esquire 1 Wyoming Avenue Big City, PA 00001 Counsel to Plaintiff

58

Page 63: FEBRUARY 2020 PENNSYLVANIA BAR EXAMINATION · PENNSYLVANIA BAR EXAMINATION Essay Questions and Examiners’ Analyses and Performance Test Pennsylvania Board of Law Examiners 601 Commonwealth

EXHIBIT A Legend of the Octopus

The Legend of the Octopus is a sports tradition during Detroit Red Wings home playoff games involving dead octopuses thrown onto the ice rink. The origins of the activity go back to the 1952 playoffs, when a National Hockey League team played two best-of-seven series to capture the Stanley Cup. Having eight arms, the octopus symbolized the number of playoff wins necessary for the Red Wings to win the Stanley Cup. The practice started April 15, 1952, when Pete and Jerry Cusimano, brothers and storeowners in Detroit's Eastern Market, hurled an octopus into the rink of The Old Red Barn. The team swept the Toronto Maple Leafs and Montreal Canadiens en route to winning the championship. Since 1952, the practice has persisted with each passing year. In one 1995 game, fans threw 36 octopuses, including a specimen weighing 38 pounds (17 kg). The Red Wings' unofficial mascot is a purple octopus named Al, and during playoff runs, two of these mascots were also hung from the rafters of Joe Louis Arena, symbolizing the 16 wins now needed to take home the Stanley Cup. The practice has become such an accepted part of the team's lore, fans have developed various techniques and "octopus etiquette" for launching the creatures onto the ice. Events inspired by the octopus

The octopus tradition has launched several other creature and object tossing moments: During Game 3 of the 1995 Stanley Cup Finals between the Detroit Red Wings and the New Jersey Devils, Devils fans threw a lobster, a dead fish, and other objects onto the ice. In the 2006 Stanley Cup playoffs, during the opening-round series between the Wings and the Edmonton Oilers, an Edmonton radio host suggested throwing Alberta Beef on the ice before the game. Oilers fans continued throwing steaks, even at away games, resulting in several arrests at the away cities. In the 2002–03 season, the Nashville Predators fans began throwing catfish onto their home ice in response to the Red Wings tradition. The first recorded instance occurred on October 26, 2002 in a game between the Red Wings and the Predators. Jessica Hanley, who helps clean the ice in the Gaylord Entertainment Center, told the press: "They are so gross. They're huge, they're heavy, they stink and they leave this slimy trail on the ice. But, hey, if it's good for the team, I guess we can deal with it." This tradition continued in Game 3 of the 2008 Western Conference Quarterfinals matchup between the Detroit Red Wings and the Nashville Predators when Predator fans threw four catfish onto the ice. During Game 4 of the 2007 Stanley Cup Western Conference Semifinals between the Detroit Red Wings and the San Jose Sharks, a Sharks fan threw a 3-foot leopard shark onto the ice at the HP Pavilion at San Jose after the Sharks scored their first goal with 2 minutes left in the first period. During the 2008 Stanley Cup Finals, in which the Red Wings defeated the Pittsburgh Penguins, seafood wholesalers in Pittsburgh, led by Wholey's Fish Market, began requiring identification from customers who purchased octopuses, refusing to sell to buyers from Michigan. This also took place in the lead up to the 2017 Stanley Cup Finals with markets refusing to sell catfish to Tennessee residents.

59

Page 64: FEBRUARY 2020 PENNSYLVANIA BAR EXAMINATION · PENNSYLVANIA BAR EXAMINATION Essay Questions and Examiners’ Analyses and Performance Test Pennsylvania Board of Law Examiners 601 Commonwealth

In Game 1 of the 2010 Western Conference Quarterfinals between the Detroit Red Wings and the Phoenix Coyotes, a rubber snake was thrown onto the ice after a goal by the Coyotes' Keith Yandle. In Game 2 of the 2010 Western Conference Semifinals between the Detroit Red Wings and San Jose Sharks, a small shark was tossed onto the ice with an octopus inside its mouth. In Game 3 of the 2017 Western Conference Finals between the Anaheim Ducks and the Nashville Predators, a Predators fan threw a skinned duck on the ice. In Game 1 of the 2017 Stanley Cup Finals between the Pittsburgh Penguins and the Nashville Predators, a fan threw a catfish on the ice in the second period, and was escorted out of the arena. Excerpt from https://en.wikipedia.org/wiki/Legend_of_the_Octopus, August 7, 2019 [endnotes and hyperlinks removed]

60

Page 65: FEBRUARY 2020 PENNSYLVANIA BAR EXAMINATION · PENNSYLVANIA BAR EXAMINATION Essay Questions and Examiners’ Analyses and Performance Test Pennsylvania Board of Law Examiners 601 Commonwealth

LIBRARY

61

Page 66: FEBRUARY 2020 PENNSYLVANIA BAR EXAMINATION · PENNSYLVANIA BAR EXAMINATION Essay Questions and Examiners’ Analyses and Performance Test Pennsylvania Board of Law Examiners 601 Commonwealth

888 A.2d 872 (Pa.Super. 2005)

JEREMY LOUGHRAN, Appellant,

v.

THE PHILLIES and MARLON BYRD, Appellees.

***

¶ 1 This is an appeal from an order granting summary judgment in favor of appellees. [1] Appellant claims the trial court misapplied the "no duty" rule in finding that a spectator at a major league baseball game is not owed a duty by either the team or individual player to protect against a ball thrown into the stands; and that the trial court incorrectly found that his injury was an inherent risk of attending the game. We disagree with appellant, and affirm the order of the trial court. ¶ 2 On July 5, 2003, Jeremy Loughran (appellant) attended a baseball game between the Philadelphia Phillies (Phillies) and the Florida Marlins. Appellant's Brief, at 5. At the end of the top half of the seventh inning, appellant was injured when Philadelphia centerfielder, Marlon Byrd, after catching a ball for the last out, threw the ball into the stands. Id. Appellant was treated twice at the Veterans Stadium Infirmary and later at St. Mary's Medical Center. Id. at 6. Appellant's immediate injuries included bleeding around his left eye, a concussion, facial contusions, and abrasions. [Footnote omitted.] Id. Appellant has since been treated for severe headaches, vomiting, confusion, incoherence, hallucinations, loss of balance, head and neck pain, photophobia, eye spasms, sleep disruption, and depression. Id. ¶ 3 Appellant filed the current negligence action against Byrd and the Phillies on March 8, 2004 and on March 8, 2005, the trial court granted summary judgment in favor of appellees, holding that "the applicable law clearly states that recovery is not granted to those who voluntarily expose themselves to risks by participating in or viewing an activity." Trial Court Opinion, 5/3/2005, at 1. This timely appeal follows. ¶ 4 Our standard of review of an order granting or denying a motion for summary judgment is well established:

We view the record in the light most favorable to the non-moving party, and all doubts as to the existence of a genuine issue of material fact must be resolved against the moving party. Only where there is no genuine issue as to any material fact and it is clear that the moving party is entitled to a judgment as a matter of law will summary judgment be entered. Our scope of review of a trial court's order granting or denying summary judgment is plenary, and our standard of review is clear: the trial court's order will be reversed only where it is established that the court committed an error of law or abused its discretion.

Sackett v. Nationwide Mut. Ins. Co., 880 A.2d 1243 (Pa.Super. 2005). ¶ 5 On appeal, appellant lists five separate "questions involved." See Appellant's Brief, at 4. For purposes of our review, however, they can be combined into one issue: whether the trial court's application of the "no duty" rule to this case was proper. [Footnote omitted.]

62

Page 67: FEBRUARY 2020 PENNSYLVANIA BAR EXAMINATION · PENNSYLVANIA BAR EXAMINATION Essay Questions and Examiners’ Analyses and Performance Test Pennsylvania Board of Law Examiners 601 Commonwealth

¶ 6 We must first note that appellant's claim was brought under a negligence theory. It is axiomatic to say that in order to succeed on a negligence claim, the four basic elements of duty, breach, causation, and damages must be established. Appellees moved for summary judgment on the grounds that "as a matter of law, [appellees] did not owe a duty to [appellant] to protect him from the risk of being struck by a thrown baseball while sitting in the stands, [and that] [appellant] assumed the risk of being struck by a thrown ball by sitting in an area where he knew balls could be thrown." Defendant's Motion for Summary Judgment, 1/4/2005, at ¶ ¶ 22, 23 (Docket Entry 24). ¶ 7 In explaining its application of the "no duty" rule, the trial court noted that appellant failed to show that appellees "deviated from an established custom in the game of baseball" in tossing a ball to the fans, and therefore appellant could not escape its application. Trial Court Opinion, 5/3/2005, at 5. The trial court further explained that regardless of appellant's claimed ignorance as to the possibility of a ball reaching the seats via a player's throw, he still could be said to have assumed that risk because it was an inherent risk in attending a baseball game. Id. at 4 (citing Schentzel v. Philadelphia National League

Club, 173 Pa.Super. 179, 96 A.2d 181 (1953)). ¶ 8 We think it necessary to first examine the nature of the "no duty" rule and specifically, its application on the baseball diamond. We have previously stated that "[t]he operator of a place of amusement is 'not an insurer of his patrons,' and therefore, patrons will only be able to recover for injuries caused by the operator's failure to exercise 'reasonable care in the construction, maintenance, and management of the facility.'" Romeo v. The Pittsburgh Associates, 787 A.2d 1027 (Pa.Super. 2001) (quoting Jones v. Three

Rivers Management Corp., 483 Pa. 75, 394 A.2d 546 (1978)). The "no duty" rule applies to bar a plaintiff's claims for injuries suffered as a result of common, frequent and expected risks inherent during the activity in question. Jones v. Three Rivers Management Corp., 483 Pa. 75, 394 A.2d 546 (1978). "Only when the plaintiff introduces adequate evidence that the amusement facility in which he was injured deviated in some relevant respect from established custom will it be proper for an 'inherent-risk' case to go to the jury." Id. at 550. It can be said that the "no duty" rule has evolved into a modified version of the assumption of the risk doctrine, which has been largely abolished in Pennsylvania. Romeo

v. The Pittsburgh Associates, 787 A.2d 1027 (Pa.Super. 2001). ¶ 9 Appellant first challenges the trial court's finding that his being hit by a ball thrown by the centerfielder is an inherent risk. Appellant argues that his injuries were not the result of "a throw that could in any way be construed as a common, frequent or expected part of the game." Appellant's Brief, at 13. In support of this argument, appellant offers that "he had never seen an outfielder throw a ball into the seats; that he had never seen a player throw a ball overhand into the seats from any location on the field; and that he was completely surprised by Byrd's throw into the stands, and was not expecting an outfielder to throw a ball into the crowded outfield seats after play had ended." Id. at 13. ¶ 10 Appellant correctly surmises that the application of the "no duty" rule hinges on whether the activity in question is a "common, frequent, or expected part of the game." He argues that because the third out had been made, the inning was over, and therefore Byrd's throw can neither be expected, nor even part of the game. When determining what is [a] "customary" part of the game, it is our opinion that we cannot be limited to the rigid standards of the Major League Baseball rule book; we must instead consider the actual everyday goings on that occur both on and off the baseball diamond; we must consider as "customary" those activities that although not specifically sanctioned by baseball authorities, have become as integral a part of attending a game as hot dogs, cracker jack, and seventh inning stretches. Fans routinely arrive early for batting practice in hopes of retrieving an errant baseball as a souvenir, and fans routinely clamor to retrieve balls landing in the stands via home runs or foul balls. Although not technically part of the game of baseball, those activities have become inextricably

63

Page 68: FEBRUARY 2020 PENNSYLVANIA BAR EXAMINATION · PENNSYLVANIA BAR EXAMINATION Essay Questions and Examiners’ Analyses and Performance Test Pennsylvania Board of Law Examiners 601 Commonwealth

intertwined with a fan's baseball experience, and must be considered a customary part of the game. Similarly, both outfielders and infielders routinely toss caught balls to fans at the end of an inning. ¶ 11 We note that during the particular game in question, there were at least twenty (20) occasions of a ball entering the stands. Defendant's Motion for Summary Judgment, 1/4/2005, at ¶ 13 (Docket Entry 24). At least two of those balls were thrown to fans near appellant by players. Id. at ¶ ¶ 13(f), 13(i). Appellant admits to having attended numerous baseball games in the past, and to having witnessed balls tossed into the stands on previous occasions. N.T., 10/29/2004, Oral Deposition of Jeremy Loughran, at 57-60. Regardless of appellant's current contention that he did not directly see the balls thrown into the stands by the players, our courts have held that even a first-time spectator at a baseball game is imputed with the common or "neighborhood knowledge" of the risks of the game. Schentzel v. Philadelphia

National League Club, 96 A.2d at 186. ¶ 12 Appellant also argues that if the trial court's decision stands, baseball fans could be said to have assumed the risk of injuries from "a resin bag, baseball glove, baseball bat, spiked shoe, catcher's mask, or some other object that may be intentionally thrown into the stands by a player." Appellant's Brief, at 14. We cannot agree with appellant's contention that the analysis of a spiked shoe or catcher's mask thrown into the stands would be the same as the current situation, as players are not regularly booed for failing to throw their shoes or equipment into the stands, nor are fans routinely seen clamoring or jockeying for position to retrieve a thrown shoe or mask. Likewise, as previously stated, the "no duty" rule applies only to "common, expected, and frequent" risks of the game; players do not commonly throw their spiked shoes into the stands following an out. ¶ 13 We agree with the trial court that the injuries received by appellant from actions taken by Phillies centerfielder Byrd constituted an inherent risk of the game. Countless Pennsylvania court cases have held that a spectator at a baseball game assumes the risk of being hit by batted balls, wildly thrown balls, foul balls, and in some cases bats. See Schentzel v. Philadelphia National League Club, 173 Pa.Super. 179, 96 A.2d 181 (1953); Iervolino v. Pittsburgh Athletic Co., 212 Pa.Super. 330, 243 A.2d 490 (1968). See also, Dalton v. Jones, et al., 260 Ga.App. 791, 581 S.E.2d 360 (2003) (holding that the doctrine of assumption of risk precluded recovery from the Atlanta Braves and their centerfielder when the centerfielder tossed a ball to fans in between innings, resulting in a permanent eye injury to a spectator). Even a casual baseball spectator would concede it was not uncommon for a player to toss a memento from the game to nearby fans. While appellant makes much of the manner in which the ball was thrown, [4] and warns of the slippery slope the trial court's decision could result in, he fails to establish that Byrd or the Phillies deviated from the common and expected practices of the game of baseball or acted in a manner which would take them out of the purview of the "no duty" rule. ¶ 14 Because we find that the trial court did not err in applying the "no duty" rule to the case at bar, we must affirm its grant of summary judgment. ¶ 15 Order AFFIRMED. ¶ 16 Dissenting Opinion by BENDER, J. BENDER, J., dissenting:

¶ 1 I respectfully dissent. In the present case, on July 5, 2003, Appellant attended a Philadelphia Phillies game and was seated in the outfield stands. With two outs in the Florida Marlins seventh inning, a ball was hit to the Phillies' centerfielder, Marlon Byrd. Byrd caught the fly ball that ended the Marlins inning

64

Page 69: FEBRUARY 2020 PENNSYLVANIA BAR EXAMINATION · PENNSYLVANIA BAR EXAMINATION Essay Questions and Examiners’ Analyses and Performance Test Pennsylvania Board of Law Examiners 601 Commonwealth

and, as one would expect after recording the final out of an inning, Byrd began running off the field toward the Phillies' dugout. However, after Byrd took a few strides he turned and threw the ball, overhand, into the stands where it struck a spectator in the face and, allegedly caused considerable injury. The evidence is clear that the ball was not thrown into the stands accidentally while in the course of game play. Had it been, I would readily join the Majority's disposition. But, as I see it, and as I believe the law sees it, there is a difference between a ball that enters the stands on an errant throw during game play and one thrown into the stands gratuitously and outside the parameters of game play. ¶ 2 As the Majority notes, the assumption of the risk doctrine has been largely abolished in Pennsylvania. However, to be more precise, one might assert that the assumption of the risk doctrine was finally abolished, at least in most part, in the case of Hughes v. Seven Springs Farm, Inc., 563 Pa. 501, 762 A.2d 339 (2000). Prior to that time, appellate courts in the Commonwealth had considered the premise that the passage of the Comparative Negligence Act [5] had spelled the end of the assumption of the risk doctrine without definitively deciding the issue. The Supreme Court had at least twice handed down decisions purporting to abolish the doctrine due to its attendant "difficulties" and "complexities," but those cases, Howell v. Clyde, 533 Pa. 151, 620 A.2d 1107 (1993) and Rutter v. Northeastern Beaver

County School District, 496 Pa. 590, 437 A.2d 1198 (1981), failed to gain a majority of the Court. Thus, despite the presence of many detractors, the doctrine survived and was a viable defense as late as the year 2000. Bullman v. Giuntoli, 761 A.2d 566 (Pa.Super. 2000). In Hughes, the Supreme Court, without noting that Howell lacked precedential value, cited that case nonetheless in announcing that the doctrine had been abolished except for statutory exceptions. One such statutory exception related to operators of ski slopes, which was of particular relevance in Hughes as the defendant there was the operator of a ski resort located in the Laurel Mountains of southwestern Pennsylvania. Since Hughes was a unanimous decision, it now appears safe to state that the assumption of the risk doctrine has, in fact, finally been abolished in Pennsylvania. Or has it? ¶ 3 A year after Hughes announced that the doctrine of assumption of the risk had been abolished as a common law defense, a panel of this Court essentially resurrected the essence of the assumption of the risk doctrine, at least as it applied to professional baseball clubs, in Romeo v. Pittsburgh Associates, 787 A.2d 1027 (Pa.Super. 2001). The panel accomplished this by simply applying the so called "no duty" rule set forth in Jones v. Three Rivers Management Corp., 483 Pa. 75, 394 A.2d 546 (1978), a case decided under law existing prior to the passage of the Comparative Negligence Act and before there was any movement toward abolishing the doctrine of assumption of the risk. ¶ 4 In Romeo, a panel of this Court concluded that a woman who had been struck by a batted ball at a Pittsburgh Pirates game had, as a matter of law and not as a matter of a failure of proof, no cause of action against the Pittsburgh Pirates Baseball Club. In reaching the conclusion that the plaintiff had not stated a cause of action upon which recovery could be granted, the panel concluded that the Pirates owed no duty to protect the plaintiff from the risk of being struck by a batted ball. On its face, this statement seemingly differed from the conclusion that the plaintiff, in going to the game, had "assumed the risk" of being struck by a batted ball. However, the panel's discussion of the law supporting this decision invoked many terms familiar to readers of assumption of the risk cases. Indeed, the panel cited a passage from Carrender v. Fitterer, 503 Pa. 178, 469 A.2d 120 (1983), in which Chief Justice Roberts noted that the assumption of the risk doctrine is merely another way of stating the fact that the possessor/operator owes no duty to an invitee. [6] Although drawing the parallel between the two "rules" and seemingly acknowledging that the difference is merely a matter of phraseology, [7] the Romeo decision proceeded as if the so called "no duty" rule had survived abolishment in Hughes. I would submit that this is a dubious conclusion. [8]

65

Page 70: FEBRUARY 2020 PENNSYLVANIA BAR EXAMINATION · PENNSYLVANIA BAR EXAMINATION Essay Questions and Examiners’ Analyses and Performance Test Pennsylvania Board of Law Examiners 601 Commonwealth

¶ 5 Nevertheless, putting aside the ability of a panel of this court to effectively reinstate a doctrine that has been abolished by the Supreme Court, the more dangerous aspect of today's decision is the extension of the rule the Majority provides by today's holding. Assuming the so called "no duty" rule set forth in Jones survived Hughes, it appears undisputed by all parties that had Appellant been sitting low in the stands behind third base and struck by an overthrown ball thrown by an outfielder intending to throw out a runner at third, he would have no cause of action as he is presumed to have assumed the risk of being hit by such a ball, the ball having entered the stands in the course of normal play. Stated in the guise of the no duty rule, neither The Phillies Baseball Club nor Marlon Byrd would have a duty to ensure that a throw made in the course of game play would not stray from its intended target and strike a fan seated in the seats near play. Imagine the consequences of a contrary rule. ¶ 6 If a baseball player owed a duty of care to ensure that a thrown or batted ball would not strike a patron, he might hesitate in attempting to throw out a runner trying to stretch a double into a triple for fear of the ball "sailing" on him and striking a patron. Similarly, a player might think twice about attempting to hit an inside pitch for fear of pulling it into the stands and smacking a spectator, and a batter might look at a called strike on the outside corner as opposed to "protecting the plate" because he knows from experience that hitting a pitch on the outside corner can lead to a screaming foul in the direction he is facing. Of course, it would be ridiculous to impose such a duty of care upon a player participating in a sporting event. The game of baseball would be forever changed. Similarly, if a duty was imposed upon the baseball club to protect those patrons seated in all portions of the stands from a thrown or batted ball, some form of netting or barrier would need to be erected to protect fans, which would diminish the view of all patrons, even where the risk of getting struck by a ball was remote. [9] ¶ 7 But Appellant was not struck by a ball thrown in the course of game play. Appellant was struck by a ball thrown into the stands after play had ended for the half-inning. The Majority seemingly finds no distinction between a ball thrown in play and one thrown outside of play. Apparently, in the Majority's view, a thrown ball is a thrown ball. Unlike the Majority, I believe there is a distinction between a ball thrown within the confines of the game and one thrown for a purely gratuitous purpose. I also believe the law recognizes this distinction. In fact, Jones, the case Romeo so heavily relied upon, itself made a distinction between the circumstances under which a fan was struck by a batted ball. ¶ 8 In Jones a patron attending a Pittsburgh Pirates game on the opening day of Three Rivers Stadium was struck by a ball hit during batting practice while she was standing in a concourse that encircled the stadium. Three Rivers Stadium, which is no longer in existence, contained two concourses. One concourse housed restrooms and concession stands, while another concourse contained a walkway located back behind right field and had large openings from which patrons could peer out at the action on the field. Jones was walking along this right field walkway and had stopped to peer out at the playing field. Jones then turned and began to head back to the concession area when she heard a warning to "watch." When Jones turned back toward the field she was struck by a ball hit during batting practice. Jones sued the Pittsburgh Athletic Company, the owner of the Pirates, and Three Rivers Management Corporation, which managed Three Rivers Stadium, in negligence and prevailed in a jury trial. On appeal, we reversed concluding that Jones had failed to prove negligence on the part of the defendants. However, when appealed to the Supreme Court, the Supreme Court reversed our decision and reinstated the jury's verdict. ¶ 9 In discussing the no-duty rule, the Court stated:

The central question, then, is whether appellant's case is governed by the no-duty rule applicable to common, frequent and expected risks of baseball or by the ordinary rules

66

Page 71: FEBRUARY 2020 PENNSYLVANIA BAR EXAMINATION · PENNSYLVANIA BAR EXAMINATION Essay Questions and Examiners’ Analyses and Performance Test Pennsylvania Board of Law Examiners 601 Commonwealth

applicable to all other risks which may be present in a baseball stadium. To settle this question, we must determine whether one who attends a baseball game as a spectator can properly be charged with anticipating as inherent to baseball the risk of being struck by a baseball while properly using an interior walkway.

Jones, 394 A.2d at 551. The Court ultimately concluded that the answer to this second question was "no." Thus, in the minds of the Jones Court, not all batted balls were to be regarded in the same manner, and it was necessary to consider the how, where and when of being struck by the batted ball before declaring that the operator owed no duty to protect the patron from that occurrence. ¶ 10 A careful reading of Jones, reveals that the no-duty rule applies not just when one's injury is caused by a risk inherent to the activity, but also when the risk in question is necessary to the activity. This point is illustrated when the Jones Court stated: "[m]ovies must be seen in a darkened room, roller coasters must accelerate and decelerate rapidly and players will bat balls into the grandstand." Id. at 550-51. However, as Jones put it, "even in a 'place of amusement' not every risk is reasonably expected. The rationale behind the rule that the standard of reasonable care does not impose a duty to protect from risks associated with baseball, naturally limits its application to those injuries incurred as a result of risks any baseball spectator must and will be held to anticipate." Id. at 551. The qualification of the above sentence with the words "will be held to anticipate" reflects that not all risks associated with an activity will be immunized from liability. Reading both passages together, it can be seen that the risks that will be immunized are those that are not only inherent to the activity, but those necessary to preserve the essential nature of the activity. ¶ 11 I do not doubt that Marlon Byrd threw the ball that hit Appellant without malicious intent. It also appears to be true that there is a custom of tossing a ball into a stand as a means of providing a souvenir to fans. I am willing to accept that this custom is in place. However, I am unwilling to accept the premise that simply because the custom is commonplace, the commonality of the custom provides blanket immunity from the way it is carried out. The majority concedes that the ball that struck Appellant was not thrown within the confines of the actual game. Nevertheless, the Majority concludes that because ball players routinely toss caught balls to fans at the end of an inning such practice is now "inextricably intertwined with the baseball experience." Majority Opinion at ---, 888 A.2d at 876. Because the Majority finds this practice inextricably intertwined with the baseball experience, they now bestow a shroud of immunity to the practice. This conclusion is erroneous for two reasons. ¶ 12 First, the Majority's reliance upon common practice to legitimize a finding of no duty contradicts a statement made in Jones. There the court provided the following caution: "The question of whether a risk is properly to be anticipated cannot be answered by looking to whether exposure to such risks is customary in the trade. This would permit defendant/stadium operators to avoid liability for universally prevalent negligent conditions, an undesirable result." Id. at 551 n. 6. Thus, the mere fact that baseball players frequently toss balls into the stands does not dictate the conclusion that a patron will be deemed to have assumed the risk of being struck by a ball purposely thrown into the stands or that no duty exists as to the manner in which the custom is carried out. ¶ 13 Secondly, imposing a duty of care into the practice of gratuitously providing a souvenir baseball to a lucky fan in no way detracts from or endangers the game being played on the field. As such, it is not necessary for the preservation of the activity to immunize the operator against the risk in question. Referring again to the quote from Jones, if movie houses are made to lighten the theatres so that no one trips, the movie-going experience would be greatly diminished if not destroyed. If amusement parks are

67

Page 72: FEBRUARY 2020 PENNSYLVANIA BAR EXAMINATION · PENNSYLVANIA BAR EXAMINATION Essay Questions and Examiners’ Analyses and Performance Test Pennsylvania Board of Law Examiners 601 Commonwealth

made to design roller coasters so as to eliminate all jerkiness and smooth out all changes in direction they would no longer be capable of being classified as "thrill rides" and the word "amusement" might be deleted from the term "amusement parks." But if baseball players, and their employers, are charged with exercising reasonable care in the practice of providing souvenir balls to patrons, the "Fall Classic" will remain a classic sporting contest and all those regular season and playoff games preceding it would still be played in a manner consistent with Abner Doubleday's original intent. ¶ 14 It is important to dissect the Majority's exact holding to understand the ramifications of that holding. The Majority does not hold that a duty existed to exercise ordinary caution in the practice of tossing a souvenir ball into the stands at the end of an inning, but there was a lack of evidence of a breach of that duty. The Majority holds that because baseballs routinely fly into the stands, and because it could be considered a custom for ball players to gratuitously toss a ball into the stands after actual game play has ended, The Phillies baseball club and Marlon Byrd owed "no duty" to a spectator in the stands with respect to the practice of tossing a ball into the stands. What this means is that now, absent an actual positive intent to injure, the practice of throwing a ball into the stands can be undertaken with complete immunity and in complete disregard for prudence. Now, neither context nor the actual manner in which a ball is thrown into the stands is material as no duty is owed to a spectator with respect to this practice. Extending the Majority's holding to a logical end, a baseball player can, from a mere few feet from the stands, wind up and throw the ball as hard as he desires into the stands or in a showboating manner whip a ball around his back and, because no duty is owed to a spectator, he would be immune from liability should it predictably strike a fan and cause injury. ¶ 15 Additionally, given the Majority's rationale that the practice of providing souvenirs to fans invokes no duty because the practice is commonplace at games, where would this rationale end? Another common practice at ball games is the hotdog or t-shirt launch. Both promotions/practices involve providing lucky fans with either a souvenir or a treat. The hotdog launch typically involves loading a foil wrapped hotdog into a cannon-like propulsion device and launching it to waiting fans located in the stands. The t-shirt toss can be accomplished the same way, but is sometimes accomplished by employing a large slingshot. Given the Majority's analysis, because these entertaining sideshows to the game are common place, when a spectator attends a game he assumes the risk of being struck by a hotdog or t-shirt propelled from one of these devices. Also, because these events could be deemed to have become inextricably intertwined with the baseball experience, the baseball clubs and those executing the giveaways have no duty to spectators in conducting them. Of course, this would mean that if one of those executing the hotdog launch imprudently aimed at spectators seated a couple of rows into the stands they would be immune if a spectator lost an eye after getting hit nearly point blank by a foil wrapped hotdog. ¶ 16 In my view, since the act of tossing a ball to fans as a souvenir is extraneous to the game and not necessary to the playing of the game, a spectator does not "assume the risk" of being struck by a ball entering the stands for this purpose, nor is there any valid reason in law or policy to extend the immunity of the "no duty" rule to this practice. Rather, if a baseball player wants to go beyond the confines of the game and provide a gratuitous souvenir to a fan, he should be charged with the obligation of doing it in a reasonably safe and prudent manner. Here, there is certainly evidence from which a factfinder might conclude that the manner in which Byrd threw the ball into the stands was imprudent. [10] Thus, a question of material fact remains and the motion for summary judgment should have been denied and the case should have proceeded to trial. Notes:

68

Page 73: FEBRUARY 2020 PENNSYLVANIA BAR EXAMINATION · PENNSYLVANIA BAR EXAMINATION Essay Questions and Examiners’ Analyses and Performance Test Pennsylvania Board of Law Examiners 601 Commonwealth

[1] Because the summary judgment motion and all corresponding filings were made by both Marlon Byrd and the Philadelphia Phillies, we will refer to them collectively as "appellees." [4] Throughout his brief to this Court, appellant uses the words "forcefully," "with sufficient force," "overhand," "arbitrarily," "unexpectedly," "intentionally," and "carelessly" to describe the manner in which Byrd threw the ball into the stands. There was no testimony regarding the force at which the ball injuring appellant was thrown, nor how it compared to the other balls reaching the stands; and it does not appear that Byrd's intent to throw the ball as a souvenir was ever questioned. Additionally, appellant admittedly did not see the ball as it was thrown, and could not testify as to it being thrown overhand or underhand. N.T., 10/29/2004, Oral Deposition of Jeremy Loughran, at 57-60. [5] 42 Pa.C.S. § 7102(a)-(b). [6] The passage from Carrender reads, in relevant part:

It is precisely because the invitee assumes the risk of injury from obvious and avoidable dangers that the possessor owes the invitee no duty to take measures to alleviate those dangers. Thus, to say that the invitee assumed the risk of injury from a known and avoidable danger is simply another way of expressing the lack of any duty on the part of the possessor to protect the invitee against such dangers.

Id. at 125. [7] The Romeo panel stated: "The 'no-duty' rule set forth in Jones clearly incorporates the concept of assumption of the risk utilized in earlier cases." Romeo, 787 A.2d at 1031. [8] That recovery for injury resulting from being struck by a batted ball has been most often precluded on the basis of assumption of the risk can be seen from the Supreme Court's quotation of the Restatement of Torts in summarizing the various types of assumption of the risk cases. The quoted passage follows:

The Restatement Second of Torts, § 496A, summarizes the general principle of assumption of the risk as follows: "A plaintiff who voluntarily assumes a risk of harm arising from the negligent or reckless conduct of the defendant cannot recover for such harm." In practice, the doctrine was more complicated. The comment to the Restatement notes that the doctrine has been used by the courts "in at least four different senses, and the distinctions seldom have been made clear." Id. § 496A comment C. [The comment summarizes one use of the doctrine by courts] as follows: 2. A second, and closely related, meaning is that the plaintiff has entered voluntarily into some relation with the defendant which he knows to involve the risk, and so is regarded as tacitly or impliedly agreeing to relieve the defendant of responsibility, and to take his own chances. Thus a spectator entering a baseball park may be regarded as consenting that the players may proceed with the game without taking precautions to protect him from being hit by the ball. Again the legal result is that the defendant is relieved of his duty to the plaintiff.

Hughes, 762 A.2d at 341. It appears clear that the Restatement regards the facts of Romeo to fall within the assumption of the risk doctrine which Hughes indicates has been abolished in Pennsylvania, save for

69

Page 74: FEBRUARY 2020 PENNSYLVANIA BAR EXAMINATION · PENNSYLVANIA BAR EXAMINATION Essay Questions and Examiners’ Analyses and Performance Test Pennsylvania Board of Law Examiners 601 Commonwealth

statutory exceptions. Hughes continued on with a Jones no-duty analysis because the legislature had specifically reserved assumption of the risk as to ski slope operators. Thus, there is no basis for reading the Supreme Court's application of the Jones no-duty analysis to the facts of Hughes as indicating that the no-duty rule had continuing viability in other contexts. [9] Notably baseball clubs do provide protective screens between home plate and the seating area immediately behind home plate, presumably to protect patrons seated there. Query: if baseball clubs owe no duty to patrons to protect them from being struck by a batted ball can the baseball clubs remove the protective netting without fear of being sued? Can the maintenance crew ignore a rip in the netting and fix it at their leisure knowing all along that the net is provided not out of any duty but gratuitously? [10] Although it does not appear that the Majority's analysis is premised upon a lack of evidence, the Majority seems to hedge its position some by asserting in footnote 4 that there was a lack of evidence that Byrd threw the ball in a forceful manner. I cannot agree. Although Appellant was not watching Byrd when Byrd threw the ball that struck him, if Appellant's allegations are true, the ball struck Appellant with sufficient force to break his eyeglasses, bloody his face and cause a concussion. Thus, basic physics indicates that the ball was thrown with considerable force. Moreover, a statement of Appellant's companion indicates that Byrd threw the ball hard in overhand fashion.

70

Page 75: FEBRUARY 2020 PENNSYLVANIA BAR EXAMINATION · PENNSYLVANIA BAR EXAMINATION Essay Questions and Examiners’ Analyses and Performance Test Pennsylvania Board of Law Examiners 601 Commonwealth

Question No. PT: Examiner’s Analysis

The applicant is assigned to draft an opinion letter to a client providing a complete analysis of why the no-duty rule would apply and would not apply, and coming to a conclusion indicating whether or not the client may rely upon that rule as a defense to the claims in the attached complaint. Formatting 1 Point

Following directions concerning formatting is an important skill of every lawyer. The applicant is expected to follow the directions in the assignment memorandum to draft a client opinion letter. Negligence 2 Points

Ms. Fendick brought a claim of negligence against The William Penns, Inc. (The Penns). Plaintiff’s

Complaint

It is axiomatic to say that in order to succeed on a negligence claim, the four basic elements of duty, breach, causation, and damages must be established. Loughran v. The Phillies, et al.

Ms. Fendick included in her complaint averments concerning duty (Plaintiff’s Complaint at 14, 18, 21); breach (Plaintiff’s Complaint at 22, 23, 24); causation (Plaintiff’s Complaint at 25, 26, 27); and damages (Plaintiff’s Complaint at 12, 13). Ms. Fendick’s complaint alleges three purported breaches of The Penns’ purported duty to keep her safe

from harm: a. The failure to prevent her from being injured when the fan threw the octopus toward the

ice; b. The failure to prevent the player from throwing the octopus back into the stands; and c. The failure to take measures to ensure that fans would not throw objects onto the ice.

Plaintiff’s Complaint

On its face, Ms. Fendick’s complaint states valid claims for negligence resulting from the

aforementioned alleged breaches. Plaintiff’s Complaint

No-Duty Rule 6 Points

Owners and operators of a place of amusement, such as a hockey arena, may have a valid defense to claims by an injured spectator at a sporting event, such as a hockey game. Loughran v. The Phillies, et

al.

It must be determined whether The Penns, as the owner and operator of the arena in question, “deviated

from an established custom in the game” of hockey when the fan threw the octopus onto the ice, and when the player threw the octopus back over the glass. Loughran v. The Phillies, et al.

Regardless of Ms. Fendick’s knowledge or ignorance of the practice of throwing objects onto the ice during hockey games, she may be said to have assumed the risk of being struck by an object because it was an inherent risk in attending a hockey game. Loughran v. The Phillies, et al.

71

Page 76: FEBRUARY 2020 PENNSYLVANIA BAR EXAMINATION · PENNSYLVANIA BAR EXAMINATION Essay Questions and Examiners’ Analyses and Performance Test Pennsylvania Board of Law Examiners 601 Commonwealth

Such an analysis, as applied to a place of amusement, is appropriately viewed in light of the no-duty rule. The no-duty rule states “[t]he operator of a place of amusement is 'not an insurer of his patrons,' and

therefore, patrons will only be able to recover for injuries caused by the operator's failure to exercise 'reasonable care in the construction, maintenance, and management of the facility.'" Loughran v. The

Phillies, et al.

The no-duty rule applies to bar a plaintiff's claims for injuries suffered as a result of common, frequent, and expected risks inherent during the activity in question. Loughran v. The Phillies, et al.

"Only when the plaintiff introduces adequate evidence that the amusement facility in which he was injured deviated in some relevant respect from established custom will it be proper for an 'inherent-risk' case to go to the jury." Loughran v. The Phillies, et al.

The application of the no-duty rule hinges on whether the activity in question is a “common, frequent, or

expected part of the game.” Loughran v. The Phillies, et al.

When determining what is a “customary” part of the game, the court is not limited to the formal rules of the game itself; the court may also consider as customary the everyday goings on that occur both on and off of the field of play, so to speak. Loughran v. The Phillies, et al.

Although not technically part of the game, those activities that have become inextricably intertwined with a fan’s experience must be considered a customary part of the game. Loughran v. The Phillies, et

al.

Argument in Favor of Applying the No-Duty Rule 5 points

The no-duty rule is not limited to baseball; rather it applies to “operators of a place of amusement.”

Loughran v. The Phillies, et al. Therefore, the no-duty rule should apply to a professional hockey venue. Because the no-duty rule applies to The Penns’ hockey arena, Ms. Fendick may only recover for injuries caused by The Penns’ “failure to exercise ‘reasonable care in the construction, maintenance, and

management of the facility.’” Loughran v. The Phillies, et al.

Ms. Fendick has not alleged a failure of reasonable care in the construction or maintenance of the facility in which she was injured. Plaintiff’s Complaint. The application of the no-duty rule hinges on whether the activity in question is a “common, frequent, or

expected part of the game.” Loughran v. The Phillies, et al.

Throwing an octopus onto the ice at Detroit Redwings games has become such an accepted part of the team’s lore, that fans have developed techniques and “octopus etiquette” for launching the creatures

onto the ice. Exhibit A to Plaintiff’s Complaint. The “octopus tradition” dates back to 1952. Id. Furthermore, the “octopus tradition” has inspired fans at numerous other hockey venues to throw

various objects onto the ice in support of their team. Exhibit A to Plaintiff’s Complaint

72

Page 77: FEBRUARY 2020 PENNSYLVANIA BAR EXAMINATION · PENNSYLVANIA BAR EXAMINATION Essay Questions and Examiners’ Analyses and Performance Test Pennsylvania Board of Law Examiners 601 Commonwealth

Thus, for more than 65 years, hockey fans have been throwing objects onto the ice in a show of support for their teams, and the tradition broadened over the past seventeen years to cover numerous arenas. Exhibit A to Plaintiff’s Complaint

Arguably, the “octopus tradition” has become a common, frequent, and expected part of professional

hockey in the same manner that a major league baseball player throwing a ball into the stands has become inextricably intertwined with a fan’s baseball experience. As a result of the inextricable intertwining of the “octopus tradition” in professional hockey, arena

operators should be immune from suit by patrons who are accidentally struck by objects being thrown onto the ice during hockey games. Ms. Fendick knew that it was common, frequent and expected that objects could fly into the stands, as evidenced by her intentional purchase of a ticket for a seat immediately behind the protective glass. Plaintiff’s Complaint Knowing of the potential for objects to fly into the stands, Ms. Fendick nonetheless proceeded to purchase the ticket and attend the hockey game. Plaintiff’s Complaint “A Plaintiff who voluntarily assumes a risk of harm arising from the negligent or reckless conduct of the

defendant cannot recover for such harm.” Loughran v. The Phillies, et al.

The Loughran Court concluded that throwing a baseball into the stands as a souvenir was not an actual part of the game of baseball, but that it was so inextricably intertwined with the game so as to be a customary part of attending the game. Loughran v. The Phillies, et al.

Similarly, the throwing of objects, such as an octopus, at a hockey game is also inextricably intertwined with the sport of hockey so as to make it customary. Thus, when a player throws an object from the ice into the stands so that it will not freeze to the ice and interfere with play (Plaintiff’s Complaint), arena owners should enjoy the same protections afforded under the no-duty rule. Ms. Fendick knew of the risk of objects being thrown in the stands at hockey games, and throwing objects is a common, frequent and expected part of hockey games, so the no-duty rule will apply and provide a defense to The Penns under the no-duty rule. Argument Against Applying the No-Duty Rule 5 points

Although octopuses have been thrown at Detroit Red Wings games since 1952, the practice of throwing objects at other arenas did not really begin to spread to other facilities until 2002-2003, with most of that activity occurring during the Stanley Cup semi-finals and finals. Exhibit A to Plaintiff’s Complaint

Such a recent and limited undertaking of throwing objects onto the ice at hockey games cannot be considered to be a common, frequent, or expected risk as required by the no-duty rule. Loughran v. The

Phillies, et al.

Further, this was not a Detroit Red Wings game; nor was it a Stanley Cup semi-final or finals game. It was a regular season game of The Penns. Complaint.

73

Page 78: FEBRUARY 2020 PENNSYLVANIA BAR EXAMINATION · PENNSYLVANIA BAR EXAMINATION Essay Questions and Examiners’ Analyses and Performance Test Pennsylvania Board of Law Examiners 601 Commonwealth

The throwing of an octopus onto the ice at a Penns’ game, when their mascot is a man dressed in a Quaker outfit (see Plaintiff’s Complaint), would not likely be an expected part of the game. The throwing of the octopus back into the stands by a player is an equally unexpected occurrence. While it certainly was not done with ill-will, it was also not done in order to give a fan a keepsake from the game, such as an outfielder throwing a ball into the stands after the third out, as in Loughran v. The

Phillies, et al.

The Penns should have instructed its players to leave objects thrown onto the ice for arena personnel to clean up, or at least not to throw the objects back into the stands, which could injure an unsuspecting fan. Ms. Fendick intentionally purchased a ticket close to the glass so that she would not be struck by a hockey puck or equipment that flew into the stands – an expected occurrence at a hockey game. In fact, she paid a premium for that safety. Plaintiff’s Complaint. Obviously The Penns foresaw these occurrences as a danger, and took action to protect fans by installing the glass. However, because it was not a common or expected occurrence, it was not foreseeable to Ms. Fendick that she would be struck from behind by an octopus thrown by a fan, or by the same octopus being thrown back over the glass by a player on the ice, as alleged in Plaintiff’s Complaint. The no-duty rule applies not just when one’s injury is caused by a risk inherent to the activity, but also when the risk in question is necessary to the activity. Loughran v. The Phillies, et al.

Throwing an octopus from the stands onto the ice, and throwing an octopus from the ice into the stands is not necessary to the activity, or the playing of a hockey game. In most stadiums, objects thrown onto the ice are cleaned up by stadium personnel instead of players (Exhibit A to Plaintiff’s Complaint), making this incident even less common, frequent and expected, where a player threw the octopus back into the stands (Plaintiff’s Complaint). Having been injured when struck in the back of the head by the octopus and smashing her head off of the glass (Plaintiff’s Complaint), Ms. Fendick was in no position to even see the octopus being thrown back over the glass by the player and was not in a position to protect herself from being further injured. Throwing an octopus is not an inherent part of a hockey game. While one may expect a puck, or some piece of equipment to occasionally make its way over the protective glass, the presence of an octopus has nothing to do with the rules of hockey or the game of hockey and, thus, is not an inherent part of the game. Loughran v. The Phillies, et al. Even if a court was to consider fans throwing objects onto the ice to be customary and, as a result a common, frequent and expected part of a hockey game, there are no facts in the materials provided that demonstrate that a player throwing objects into the stands is a common, frequent or expected part of the game. Thus, even if the no-duty rule applies to the act of the fan throwing the octopus, it should not apply to the player’s act of throwing the octopus back into the stands. Preventing fans from throwing objects onto the ice, and preventing players from intentionally throwing objects into the stands would not have an adverse effect on the quality of play during the game, and would not affect how players play the game.

74

Page 79: FEBRUARY 2020 PENNSYLVANIA BAR EXAMINATION · PENNSYLVANIA BAR EXAMINATION Essay Questions and Examiners’ Analyses and Performance Test Pennsylvania Board of Law Examiners 601 Commonwealth

Arguably, although not phrased as such, Ms. Fendick may have alleged a defect in the management of the facility by alleging a failure to prevent fans from bringing objects into the arena that could be thrown onto the ice during hockey games at Defendant’s facilities, and by failing to institute safeguards against

fans throwing objects toward the ice at hockey games at Defendant’s facilities. If the court accepts such an argument, the no-duty rule would not apply. Finally, the application of the no-duty rule stands as a revival of the theory of assumption of the risk. Loughran v. The Phillies, et al. Assumption of the risk is a legal theory that has been largely abolished in Pennsylvania. Loughran v. The Phillies, et al. Thus, the court should not apply the no-duty rule in the instant matter. Conclusion 1 Point

Applicants are asked to come to a conclusion whether or not the no-duty rule will be available to The Penns as a defense to the negligence claims contained in Ms. Fendick’s complaint.

75